Sie sind auf Seite 1von 89

2009/2010

2009
Long Case Examination for Phase III Medical Students (Obstetric Cases)
List and Answer to Commonly Asked Questions by Lecturers

Muhamad Naim B Ab Razak


University Science Malaysia
jacknaim@gmail.com
www.jacknaimsnotes.blogspot.com

Open Notes to My friends

In the name of Allah,


The Most gracious and the Most Merciful:
Assalamualaikum wbt,
May Peace and Blessing of GOD be upon all of you

Dear friends,
I would like to take this opportunity to thank you for inspiring me a lot during our journey in
medical school.
This notes is my way of replying your kindness and favor in helping me to survive the
challenging life of medical school. Lot of cry and tears, but yet we still able to laugh together!
All thanks to the seniors who have been doing a great job by compiling the entire
commonly asked question during an exam. I just add some spice to their effort by providing
an answer to the questions through this little book.
If there is any mistake in this book, do not hesitate to inform me. InsyaALLAH I will try my
best to correct it and updating this book.
Whatever you read in this book, please double check with current management and ask
opinion from lecturers.
This book mainly reserves to be used for last minutes revision and not as reference. Please
keep on reading text book and enhance your knowledge through journals. It is our duty as
a Muslim to keep on updating our knowledge
Hopefully, all of us may become a great and outstanding Muslim doctor someday. Pray for
me that I may pass my undergraduate study and successfully pursuit my dream to
become an emergency specialist.
Sincerely yours,

Jacknaim

Oath Of USM Medical Students Graduation Day

In the name of God,


We seek from you:
The ability to be truthful, honest modest,
Merciful and objective in our dealings
The fortitude to admit our mistakes,
to amend our ways and to forgive
The wisdom to comfort and counsel all our patients
Towards well being, peace and harmony regardless of their
Social status, race and religion
The ability to understand that our profession is sacred,
Dealing with your most precious gifts of life and intellect
We promise to devote our lives in serving mankind,
Poor or rich,
Literate or illiterate,
Irrespective of race and religion
With Patience and tolerance,
With virtue and reverence,
With knowledge and vigilance,
And with Your love in our heart

The Oath Of the Muslim Doctor

I swear by God ...The Great


To regard God in carrying out my profession
To protect human life in all stages and under all circumstances,
doing my utmost to rescue it from death, malady, pain and anxiety. .
To keep peoples' dignity, cover their privacies and lock up their secrets...
To be, all the way, an instrument of God's mercy,
Extending my medical care to near and far,
Virtuous and sinner and friend and enemy.
To strive in the pursuit of knowledge and. harnessing it for the benefit
but not the harm of Mankind.
To refer my teacher, teach my junior,
And be brother to members of the Medical Profession.
Joined in piety and charity.
To live my Faith in private and in public,
Avoiding whatever blemishes me in the eyes of God,
His Apostle and my fellow Faithful.
And may God be witness to this Oath.

Long Case Examination for Phase III Medical Students


University Science Malaysia

Important and Common Cases Needs to be Covered in Obstetric Section.


1) Normal labour
2) False labour
3) Unsure of Date
4) Induction of Labour
5) Caesarian Section
6) Pregnancy Induce Hypertension
7) Pre eclampsia
8) Hypertension in Pregnancy
9) Diabetes Mellitus in pregnancy
10) Gestational Diabetes Mellitus
11) Oligohydramnios
12) Polyhydramnios
13) Reduced Fetal Movement
14) Threatened pre term labour
15) Premature birth.
16) Post Date
17) Post Term
18) PPROM
19) PROM
20) Placenta previa
21) Unstable lie
22) Breech presentation
23) Multiple pregnancy
24) Heart disease in pregnancy
25) Anemia in pregnancy
26) Fibroids
27) Anti phospholipids syndrome
28) Teenage pregnancy

And We have enjoined on man (to be good) to his parents: in travail upon travail did his
mother bear him, and in years twain was his weaning: (hear the command), "Show
gratitude to Me and to thy parents: to Me is (thy final) Goal [Q31:14]]

Long Case Examination for Phase III Medical Students


University Science Malaysia

Anti tetanus toxoid


Type of immune (page 36)
When to give (page 36)
Anemia in pregnancy (page 44,45)
Breech presentation (page 36)
Causes and complication
Mode of delivery
Candidosis
Drug (page 24)
Caesarian section (page 9)
Anterior abdominal wall layer (page 10)
Impending scar rupture (page 10)
Preparation pre op and post op (page 9)
Pfannensteil scar (page 32)
Trial of scar (page 9)
Cervix
Normal cervical length (page 3)
cervical effacement (page 1)
cervical dilatation in nulli vs multiparity
(page 1)
Cervical cerclage (page 26)
Chorioamnionitis
Common organism (page 8)
Management (page 8)
Diabetes Melitus in pregnancy
GDM (page 15)
Complication of GDM (page 16)
Diagnosis and level of sugar control (page
15)
Screening test (page 15)
Diabetogenic hormone (page 15)
Hydrocephalus (page 19)

Macrosomic baby (page 18)


Management (page 16, 17)
MOGTT, when to do (page 15)
MOGTT, Indication (page 16)
Shoulder dystocia (page 18)
Spontaneous vaginal delivery ((page 18)
Weight gain in pregnancy (page 15)
Heart disease in pregnancy (page 40, 41,
42, 43)
Aspirin, IV
Contraindication for pregnancy (page 43)
Failure (page 40)
Eisenmenger syndrome (page 41)
Warfarin (page 41)
Episiotomy
Definition (page 4)
Layer cut (page 4)
Fibroids (page 46)
Hypothyroidism (page 50)
Labour
Bishop score (page 1, page 3)
Braxton Hicks contraction (page 2)
definition of labour (page 5)
Discharging patient in latent phase of
labour (page 3)
engagement (page 4)
Show (page 5)
True vs false labour (page 2)
Management of active phase of labour
(page 2)
Mechanism of labour (page 4)
Induction of labour
definition (page 7)
Indication (page 7)

Long Case Examination for Phase III Medical Students


University Science Malaysia

Mehtod (page 7)

Ultrasound
placenta (page 6)

Oligohydramnios (page 20)


Parity
Pseudoprimid (page 5)
Grand multiparity (page 26)
Great grand multiparity (page 49)
Polyhydramnios (page 21)

Premature labour (page 24)


Definition
Premature contraction (page 25)
Management
Reduce fetal movement (page 22, 23)
Tocolytic
Fetal kick chart

Post date (page 27)


Tradisional medicine (page 23)
Post term (page 28, 29)
PPROM (page 30)
Fever
Positive findings
Management

Twin pregnancy (page 37, 38, 39)


Classification
complication
Physical examination
Twin to twin transfusion reaction

PROM (page 31)


Placenta
seperation, sign and symptoms (page 2)
Placenta previa (page 32, 33)
Prostin
Complication (page 7)
dose (page 8)
Instruction to patient (page 8)
with presence of contaction pain (page 1,
page 3)
Pregnancy induced hypertension
definition (page 11)
Essential hypertension (page 14)
Management (page 11)
Pre eclampsia (page 12)
Impending eclampsia (page 13)
Magnesium sulphate (page 13)

Unsure of date (page 7)


Neagele's rule (page 7)
Comfirmation of date
Unstable lie (page 34, 35)
Causes
Complication
Management
Mode of delivery

Uterus
support (page 4)

Long Case Examination for Phase III Medical Students


University Science Malaysia
25 years old Malay lady, G1P0 at 37W+ 2/7 are
admitted because of contraction pain but not
associated with show or leaking.
Questions
1) How do you access the favorable of
cervix?
2) What is cervical effacement?
3) Are there any differences if the cervix is
1 cm dilated in primid vs. Multipara
who presented with contraction pain at
term?
4) Can we induce the labour with Prostin
with the present of recorded contraction
pain?

Image from: Joan Pitkin et al, Obstetrics and


Gynaecology: An Illustrated colour Text

Differences if the cervix is 1 cm dilated in


primid vs. Multipara who presented with
contraction pain at term?

Answer
Cervical score
In HUSM, we used Modified Bishop Score.
Cervix is favorable if Bishop score > 5

Nulliparous women have small external os at


cervix center. In multiparous woman, cervix is
bulkier and the external os has a more slit like
appearance. Therefore, dilatation of 1 cm is
significant in primid and not in multigravida.
In multiparous, it is usually normal if cervix
dilatation is 1 cm. Diagnosis of false labour
should be made if it did not progress.
Role of Prostin in the presence of recorded
contraction pain.
Recorded contraction pain is by evidence of
CTG reading plus typical history of contracting
pain.

Mnemonics:
DiCoLePoS
(Dilatation,
consistency, length, Position and Station)
[Credited to Dr Ramli Ibrahim, HUSM]

Once it present, Prostin should never being use


as it will predispose mother to uterine hyper
stimulation and cause fetal distress to the baby.

Cervical Effacement
Cervical changes prior to onset of labour where
cervix become shorter, softer and moves from its
position in the posterior vaginal fornix towards
anterior vaginal fornix [Joan Pitkin et al,
Obstetrics and Gynaecology: An Illustrated
colour Text]

Other mode of induction of labour should be


considered.

Long Case Examination for Phase III Medical Students


University Science Malaysia
Impression: Patient is already in active phase of
first stage of labour.
Post date patient who are already in labour will
not change the management and spontaneous
vaginal delivery should be expected except there
is indication for caesarian section or
instrumental deliveries.

Case: G1P0, Post EDD, currently in labour (VE


5cm)
Question:
a) Patient is a primid, never experience
before, how are you going to ask her in
Hx whether it is a true labour.
b) How to manage this patient

General management
1) Transfer patient to Labour room
2) FBC and GSH

Notes: A Braxton Hicks contraction is a normal


irregular uterine contraction starts occurring
from fourth months of pregnancy. It acts as
preparation for uterus to contract properly later.

First stage of labour


1) Review history and problems
2) Abdominal exam and VE +ARM
3) Starts partogram
4) Review patient after 4H since cervix is
<6cm (if >6cm VE when full dilatation
is expected)
5) Monitor
a) Maternal
BP,
PR,
Uterine
contraction
b) 4H temperature
c) FHR auscultation/ CTG

True vs. False Labour pain


1) Timing of contractions
False Labor: Often are irregular and do not
get closer together
True Labor: Come at regular intervals and as
time goes on, get closer together. Contractions
last about 30-70 seconds.
2) Change with movement
False Labor: contractions stop in association
with walking or change in position.
True Labor: Contractions continue, despite
movement or changing positions

Second stage of labour


1) Leave patient for 30 minutes if no
pushing contraction. Notify MO if not
deliver after 1H of active pushing
2) Episiotomy

3) Strength of contractions
False Labor: Contractions are usually weak
and do not get much stronger (may be strong
first, then get weaker)
True Labor: Contractions steadily increase
in strength

Third stage of labour (30 Minutes)


1) Syntometrine
(Oxytoxin
5U+
ergometrine 0.5 mg) IM
2) Delivery of placenta by controlled cord
traction
3) Repairing of episiotomy wounds

4) Pain of contractions
False Labor: contractions are usually only
felt in the front of the abdomen or pelvic region
True Labor: Contractions usually start in the
lower back and move to the front of the
abdomen. Referred pain from uterus felt at the
buttock.

Management of this patient


2

Signs of placenta separation


Uterus contract and fundus become
globular and firm
Small gush of blood flow out
Lengthening of umbilical cord.

Long Case Examination for Phase III Medical Students


University Science Malaysia
28 years old Malay lady, G1P0 at 38W + 5/7
days POA was admitted because of contraction
pain. There is no show or leaking liquar.Below
is her Bishop score on admission.

Before, any decision to discharge this patient,


few measures needs to be look at.
1) If the contraction pain starts to subside
2) Pre discharge vagina examination did
not show any cervical progression
3) No Pre labour ruptures of membrane.
4) CTG has been performed and reactive
5) Baby is not in mal presentation.
6) Patient can easily come back to hospital
if anything happen.
a) Short distance
b) Access to transportation
c) People to take care of her.
7) With advice that patient must come to
hospital if any PROM or show or if the
contraction become strong and in close
intervals.

Questions
1) Comment on the Bishop score
2) What is the normal length of the cervix
in non pregnant lady?
3) If this patient requested to be
discharged, can you allow that? Support
your answer.
4) Will you induce this patient for labour?

Induction of labour
-

Bishop Score
Based on assessment on Bishop Score, patient is
already in latent phase of labour as evidence of
os is dilated. However the cervix is not favorable
for labour yet.
Normal cervix length for non pregnant lady
3.5 CM
Requested to be discharged
This patient is in the latent phase of labour. In
primid, the latent phase could be as short as one
day but may extend up to one week.
3

Induction with prostin is not indicated as


contraction is already there. It will only
increase risk for uterine hyper
stimulation and abruptio placenta.
ARM could be done if cervical
dilatation more than 3 cm.

Long Case Examination for Phase III Medical Students


University Science Malaysia
Mechanism of labour

23 years old Malay lady at 39W+3/7 POA


was transferred into ward from labour room
because of contraction pain associated with
show on the day of admission. No leaking of
liquor reported and fetal movement was
good.

Changes in position of the fetal head during


passage through the birth canal in the vertex
presentation.
(EDFIERE!)

Questions
1) Types of pelvis
2) What is engagement
3) Outline the mechanism of labour
4) What is the layer cut during the
episiotomy procedure?
5) The structures supporting the uterus.

1)
2)
3)
4)
5)
6)
7)

Engagement
Descent
Flexion
Internal rotation
Extension
External rotation
Expulsion

What is episiotomy and layer cut during the


procedures?
Episiotomy is a surgical cut that is made to the
perineum during the pushing stage of labour.
Layers of cutting:
1) Skin
2) Subcutaneous tissue
3) Vaginal mucosa
4) Bulbospongiosus muscle
5) Deep and superficial transverse perineal
muscle

Engagement
Descent of the biparietal diameter of the fetal
head below the plane of the pelvic inlet.
Clinically, if the lowest portion of the occiput is
at or below the level of the maternal ischial
spines (station 0), engagement has usually taken
place. Engagement can occur before the onset of
true labor, especially in nulliparous patients [The
John Hopkins Manual of Gynecology and
Obstetrics 3rd ed.]

Support of the Uterus


1) Tone of levator ani muscle
2) Perineal body
3) Ligaments
a. Transverse cervical or cardinal
ligament
b. Pubocervical
c. Sacrocervical

Long Case Examination for Phase III Medical Students


University Science Malaysia
36 years old Malay lady, G3P0 at 37 weeks of
pregnancy was admitted to ward after noticing
spotting blood mixing with mucous on her
underpants after waking up from sleep. The
same event occurs two times in ward. However,
there is no recorded abdominal pain.

Question
1) What is mature pseudo primid?
2) What is labour
3) Terminology for blood mixing with
mucous
4) Differential diagnosis
5) Management to this patient.

Management to this patient


1) Full history and physical examination
a) Correct dating of pregnancy
b) Elicit any risky pregnancy
c) Eliminating
the
differential
diagnosis.
d) It is important to exclude PROM as
patient at term and chorioamnionitis
could be disastrous for fetus.
2) Observation of vital sign
3) Fetal kick chart (some doctor
recommend this) and labour progression
chart (LPC)
4) Speculum examination to access the
cervix and excludes PROM, infection.
5) Assessment of fetal well being (CTG
and ultrasound)
6) Blood investigation (FBC to look for
evidence of infection)
7) Urine FEME to exclude UTI.
8) Observe the patient in wards for 2-3
days. If patient is stable and the labour
does not progress, then the diagnosis is
false labour. Patient can be safely
discharge and ask her to come back
again once the sign and symptoms of
labor starts.
9) If patient in labour, then proceed with
the management for labour

Answer

Differential diagnosis
In labour
False labour
PROM
Bleeding from PP or Placenta abruptio.
Discharge from urinary tract infection
Trauma to the perineal region.

Mature pseudo primid


Mature means age of mother > 35 years
old
Pseudo primid means patient has been
pregnant but never deliver the baby.
The term mature should alert the
doctor in carefully managing this patient
because of many complication can occur
in this age group. Furthermore, this
could be her last pregnancy
Labour
Process by which fetus is expelled from
the uterus and into the outside world.
Three stages of labour
a) 1st stage- onset of contraction till
full dilatation of cervix
b) 2nd stage- full dilatation of cervix till
delivery of fetus
c) 3rd stage- delivery of placenta
Sign and symptoms of labour includes
abdominal contracting pain, show
(discharging blood mixing with
mucous), gushing of clear fluid (liquor)

Long Case Examination for Phase III Medical Students


University Science Malaysia
Notes on U/S about placenta

Placental thickness judged subjectively

Vascularity

But if measure at midposition or cord


insertion 2-4 cm = normal

Very vascular has 2 blood supplies


Grade 0
1.Late 1st trimester-early 2nd trimester
2.Uniform moderate echogenicity
3.Smooth chorionic plate without indentations

Blood from fetus through 2 (sometimes 1)


umbilical arteries through umbilical cord
from fetal hypogastric arteries to placenta
1 umbilical vein carries blood back to fetal
left portal vein

Grade 1
1.Mid 2nd trimester early 3rd trimester (~18-29
wks)
2.Subtle indentations of chorionic plate
3.Small, diffuse calcifications (hyperechoic)
randomly dispersed in placenta

Blood from mom through branches of


uterine arteries through the myometrium
(arcuate arteries) through the basilar plate
(spiral arteries) into the placenta
The two circulations intertwine in the
placenta but do not mix

Grade 2
1.Late 3rd trimester (~30 wks to delivery)
2.Larger indentations along chorionic plate
3.Larger calcifications in a dot-dash
configuration along the basilar plate

Exchange of oxygen and nutrients occurs


over the large vascular surface area
Maternal venous channels in the placenta are
hypoechoic or anechoic spaces called
venous lakes (usually small, but can be
large)

Grade 3
1.39 wks post dates
2.Complete indentations of chorionic plate
through to the basilar plate creating
cotyledons (portions of placenta separated by
the indentations)
3.More irregular calcifications with significant
shadowing
4.May signify placental dysmaturity which can
cause IUGR
5.Associated
with
smoking,
chronic
hypertension, SLE, diabetes

Anatomy on US
Inner border of placenta against the uterine
wall has the combined hypoechoic
myometrium and interposed basilar layer =
hypoechoic band called the decidua basalis
(contains maternal blood vessels)
Outer surface abutting the amniotic fluid =
chorionic plate (chorioamniotic membrane)
= bright specular reflector

Sources:
http://www.learningradiology.com/notes/gunote
s/placentapage.htm

Long Case Examination for Phase III Medical Students


University Science Malaysia
Case: Unsure LMP/ Unsure of Date

to pregnancies at gestations greater than the


legal definition of fetal viability (24 weeks).

Questions
a) Neageles rule
b) Investigation
c) Management
d) Induction of labour
e) Complication of Prostin

Divided into mechanical (Sweep & scratch,


ARM,) and pharmacological (IV Syntocinon,
Prostin).
Others;
breast
stimulation,
relaxin,
hyaluronidase, sexual intercourse, acupuncture,
homeopathic method

Neageles rule
1) Sure of date
2) Menstrual cycle is regular of 28 days
(ovulation occur 14 days prior to the
next menses)
3) Not on any form of hormonal
contraception within 3 months
4) Not lactating within 2 months

Indication for IOL


1) Fetal
a) IUGR
b) PIH/PE
c) GDM at 38w
d) Post EDD
e) Twin at term
f) Hx of unexplained APH
g) Transverse oblique/unstable lie
h) Hemolytic disease
i) Fetal abnormality incompatible with
life (anencephaly)
2) Maternal
a) Medical disorder aggravated by
pregnancy like DM, SLE, PE, Renal
disease.
b) IUD with risk of DIC
c) Spontaneous/ PROM>24h
d) Abruption of placenta

Investigation
1) Cardiatocography (CTG) to access fetal
well being
2) Ultrasound for physical biometry of the
baby, and amniotic fluid index

Management
1) Confirmation of the date of pregnancy
a) Early ultrasound scan (<20w)
b) 1st UPT positive (6-8w)
c) Quickening
d) Uterine
size
correspond
to
pregnancy
e) Onset of signs and symptom of
pregnancy
f) Conception date
2) Bishop score (>5 is favorable)
3) Elicit any medical problem.

1)
2)
3)
4)
5)
6)

Induction of labour
An intervention designed to artificially initiate
uterine contractions leading to progressive
dilatation and effacement of the cervix and the
birth of the baby. The term is usually restricted

Complication of prostin
Failed IOL (require c-sec)
Uterine hyper stimulation
Uterine rupture.
Fetal distress.
C/I in patient with asthma/glaucoma
Abruptio placenta

Long Case Examination for Phase III Medical Students


University Science Malaysia
In woman whom deliver more than two babies
(not grand Multipara) and 1 caesarean section
scar, the dose for each cycle is 1.5 mg.

27 years old Malay lady, G2P1 at 38 weeks of


pregnancy was admitted to ward because of
PROM more than 24 hours. She was council for
induction of labour.

If labour is not progress after the second dose,


then it is considered as failed induction [NICE
Guidelines] and emergency C-sec will be done.
HUSM did not follow this guidelines and IOL
with prostin is based on clinical experience.

Question
1) Common
organism
causing
chorioamnionitis in PROM and how to
manage.
2) Dose of Prostin
3) Instruction to the patient before inserting
the Prostin

Some might consider failed induction after the


third dose, 6 hours after the second dose.
(Controversy)

Answer
Notes: Prostin is contraindicated if presence of
uterine contraction to avoid uterine hyper
stimulation.

Common organism causing chorioamnionitis


in PROM and how to manage?
1) Risk of getting infection arises after 12
hours of PROM.
2) Antibiotic prophylaxis should be given
based on common isolated organism
which is group B Streptococcus (IV
Penicillin)
3) Chorioamnionitis is more dangerous to
fetus as compared to mother
4) IOL should be suggested to the mother
if PROM > 24 hours. 90% of patient
with ruptured membrane will deliver the
baby within 24 hours.
5) If chorioamnionitis develop, patient
should be covered with antibiotic
against GBS, gram negative and
anaerobes.

Instruction to patient before inserting the


Prostin
1) Ask the patient to urinate first because
she needs to lie on bed for one hour
2) Ask the patient to lie down on bed for
one hour
3) Ask the patient to inform the doctor if
the contraction pain is strong.
4) Do CTG after one hour to access uterine
contraction and any evidence of fetal
distress
5) After one hour, do the VE to access the
cervical dilatation.
6) If cervix is more than 3 cm, remove the
residual Prostin and sent patient to
labour room.
7) If less than that, and suspect uterine
hyper stimulation, remove the residue
Prostin as well and send patient to
labour room and monitor with CTG.
KIV tocolytic agents (salbutamol). If
fetal distress, emergency cs.
8) If CTG normal, patient can regain her
activity. Recheck cervical score 6 hour
later.

Dose of Prostin
Notes: I suppositories equals to 3 mg.
In primid, we can insert 1 suppository and
access the Bishop score 6 hours later. If cervix is
favorable, then we may proceed with artificial
rupture of membrane. If not, second dose of
Prostin may be given.

Long Case Examination for Phase III Medical Students


University Science Malaysia
3) To cover the surgery
a) Consent form signed
b) Baseline blood investigation (FBC,
GSH, LFT, BUSE/Creat)
c) Blood cross match (2U Pack cell)
d) IV
ampicillin
1g
stat
for
prophylactic
e) Bladder catheterization
f) Pre med (IV Ranitidine 50mg in 10
ml by slow injection, IV Maxalon
10 mg by slow injection, Sodium
citrate 30 ml orally)
4) Anesthetist with at least one year
experience
5) Ideally use regional block except contra
indicated (major placenta previa, local
skin sepsis, severe heart disease,
coagulation disorder, severe fetal
distress, cord prolapsed, eclampsia)
6) Present of obstetrician.
7) Reduce risk of thromboembolic
phenomenon after surgery
a) Early ambulation
b) Anti embolic stocking/Flowtron
c) Anti coagulant for high risk cases.

Case: 3 Previous C-sec scars


Question
a) Investigation
b) Management
c) Post op-acute management
Trial of Scar
Notes: According to ACOG guidelines on
vaginal birth after Caeserean Section, trial of
scar is not recommended in patients at high risk
for uterine rupture. One of the contraindication
including this case.
1) Two prior uterine scars and no vaginal
deliveries
2) Previous classical or T-shaped incision
or extensive transfundal uterine surgery
3) Previous uterine rupture
4) Medical or obstetric complication that
precludes vaginal delivery
5) Inability to perform emergency cesarean
delivery because of unavailable surgeon,
anesthesia, sufficient staff, or facility

[The practical Labour Suite Management- Dr Adibah Ibrahim]

Notes: The management of this patient should


emphasize more on caesarean section and
anticipating in possibility of uterine rupture. It
also includes advice for tubal ligation. (Practice
in Malaysia to do BTL after 4 Caesarian
Section)

Post op management
1) Recovery area (one to one observation until
patient has airway control, cardio respiratory
stability and can communicate)
2) In wards (1/2hly observation RR, HR, BP,
pain and sedation) for 2H, then hourly if stable
3) Intrathecal opiods- hourly observation for RR,
Sedation and pain scores for 12h for
diamorphines and 24h for morphines)
4) For epidural opiods and patient-controlled
analgesia with opiods (hourly monitoring during
CS, plus 2h after discontinuation)
5) Post natal care (analgesic, monitor wound
healing, signs of infection)
6) consider CS complication (endometritis,
thromboembolism, UTI, urinary tract trauma)
[NICE Guidelines on Caesarian Section]

Investigation
Fetal investigation
- Ultrasound (AFI, Estimated fetal
weight, exclude placenta previa, accrete
or abruptio, biometry)
- CTG
Maternal (preparation for C-sec)
1) For patient in labour (fluid diet and T.
Ranitidine 150 mg q.d.s)
2) Patient at high risk of anesthetic( sips of
water+ IV fluid if indicated)
9

Long Case Examination for Phase III Medical Students


University Science Malaysia
4) Forceps application and breech extraction
once full cervical dilatation achieves
5) Elective caesarean section
6) Explore the genital tract after difficult or
instrumental delivery
7) Blood FBC and GSH

26 years old Malay lady, housewife, G2P1 at 38


weeks of gestation with second husband and
history of previous caesarean section was
admitted because of c-sec scar tenderness.
Questions
1) S&S of impending scar rupture
2) Management for patient come with
impending scar rupture
3) Elicit the scar tenderness on PE
4) The anterior abdominal wall layer cut
during the c-sec operation.

Once the ruptured occur


1) Secure the ABC. 02 100%, 3L/min increase
oxygenation to tissue if hemorrhage occurs.
2) 2 large bore IV line
3) Blood transfusion and shock management
4) Emergency laparatomy
5) Delivery of fetus and placenta
6) Exploration of the rupture site
a) Try to repair the lesion
b) Hysterectomy of not salvageable
7) Internal iliac artery ligation in case of broad
ligament hematoma because uterine artery is
usually retracted and difficult to be identified.
8) Vaginal repair if there is cervical tear

Uterine scar rupture


A complete uterine rupture is a tear through the
thickness of the uterine wall at the site of a prior
cesarean incision.
Patient might present with:
1. Fetal distress evidence by abnormalities
in fetal heart rate
2. Vaginal bleeding
3. Sharp onset of pain at the site of
previous scar
4. Sharp pain between contractions
5. contraction become less intense (finally
lead to atony)
6. Diminished baseline uterine pressure
7. Abdominal tenderness
8. Recession of the presenting fetal part
9. Hemorrhage
10. shock

Layer cut through caesarean section


(Pfannenstiel approach)
1) Curved transverse cut just below hair
border
a) skin
b) superficial fascia (Camper and
Scarpa)
c) Rectus sheath (contains fascia of
EO, IO and TM)
2) Vertical incision for access into lower
abdomen
a) Separation of rectus abdominis
muscle in midline
b) Dividing of the fascia transversalis
c) periperitoneal fat tissue
d) peritoneum

Management to impending scar ruptures


Management
Prophylactic management
1) Close monitoring for woman with high risk of
uterine rupture
2) Early detection of causes of obstructed labour
3) Use Oxytoxin with caution
10

Long Case Examination for Phase III Medical Students


University Science Malaysia
being, abnormal surveillance basic
blood test (BP and urine dipstick at least
3X per week, weekly PE profile and
CTG.)
6) Starts anti hypertensive when diastolic
BP > 90 mmHg
a) T. Methyldopa 250 mg tds to max
dose of 3g/day or
b) T. labetolol 100 mg tds to max 300
mg tds
7) IM dexamethasone 12 MG 12 hourly for
two doses for expectant prem delivery.

Case: PIH
Questions
a) Differential diagnosis
b) Management
c) Drugs (SE&MOA)
d) Drugs contraindicated in PIH
Definition
BP more than or equal to 140/90 mmHg in
previously normotensive patient, @ A rise in
systolic BP of > 30 mmHg or diastolic BP > 15
mmHg compared with pre-conception or first
trimester value in two recording of at least 4H
apart

In case of severe PE
1) Manage in hospitals
2) Close monitor BP 4Hly, reflex, clonus
3) Check fundus
4) Twice weekly(or more based on
severity) PE, CTG, biophysical profile
and doppler
5) Anti-hypertensive but aim for 20-25
reduction only and not normal by using
hydrallazine or labetolol

Differential diagnosis
- Chronic hypertension (long or before 20w)
- Pre eclampsia (>20W+new onset proteinuria)
- PE with superimposed chronic HPT
New onset or A) acutely worsen proteinuria, B)
sudden increase in BP, C) thrombocytopenia or
D) elevated liver enzymes after 20 week
gestation in women with pre existing HPT
- Gestational HPT (after 20w without
proteinuria)

1)

2)
3)

4)

5)

In labour
1) BP stabilization
2) Watch for fluid overload (monitor UO)
3) Seizure prophylaxis in severe PE
4) Epidural analgesic is the best
5) Oxytoxin only to augment labour.
6) Never allow woman with severe PE to
push excessively. If BP high, consider
instrumental delivery.
7) C/I to ergometrine/syntometrine in third
stage due to hypertensive effect.

Management
if detected <20W, must exclude molar
pregnancy by US and after exclusion,
being investigate for primary or
secondary HPT
If pre existing HPT during Booking,
should be managed by obs+internist
Every other day BP check at local clinic
if BP is first high during any ante natal
check up.
Investigation for PE profile (platelet
count, uric acid, serum creatinine level,
AST, urine albumin). If PE is diagnosed,
then it should be repeated once a week
If BP sustained at >100mg/ >25
increment mmHg or clinical suspicious
of IUGR, poor maternal-feternal well

Drugs contraindicated for PIH includes ACE


inhibitor and ARB as it can cause renal
dysgenesis of the baby.

11

Long Case Examination for Phase III Medical Students


University Science Malaysia
5) PE profile twice a week (severe PE) or
once a week(mild PE) compose of
a) Platelet count (decrease)
b) Uric acid (1st indication of renal
impairment)
c) Sr Creatinine level (renal function)
d) Liver enzyme, AST (liver damage)
e) Urine albumin as mention in above.
6) Clotting study if platelet < 100 x 106/l
7) Input/output Fluid Chart.
8) CTG for fetal well being.
9) Serial ultrasound measurements of fetal
size, umbilical artery Doppler and liquor
volume

Case: 41/M/F, G1P0 at 29W+2d POA


High blood pressure and proteinuria 3+
Question
a) Investigation and reason
b) Treatment plan
c) Time of delivery and why?
My impression: High blood pressure with
proteinuria could lead to Pre eclampsia which is
worrisome due to serious complication.
Therefore, PE should be ruled out first before
considering other condition that may falsely give
positive result to proteinuria like UTI
PE is defined as:
Hypertension unique to pregnancy, diagnosed
after 20W of gestation and associated with new
onset proteinuria; Eclampsia if seizure occurs.

Treatment plan
Mild PE
T. Methyldopa 250mg tds, max 3g/day or
T. Labetolol 100 mg tds, max 300mg tds
Or, Tab. Nifedipine 10 mg tds stat dose

If woman already having pre existing HPT but


after 20W she develops new onset proteinuria,
sudden increase in BP, thrombocytopenia or
elevated liver enzymes, then PE with
superimposed on chronic hypertension must
be suspected.

Severe PE
IV hydrallazine start 5mg, double if no effect
until 35mg. change drug if fails or
IV Labetolol start 10 mg, double if no effect
until max 300mg/day)

HELLP (Hemolysis, Elevated liver enzyme, low


platelet) is a variant of PE with involvement of
liver giving rise to tender epigastric pain, and
finally DIC.

1)

2)
3)
4)

** MgSo4 slow infusion 4g 10-15 minutes.


Maintenance dose IV ig/hour
When to Deliver
1. Delivery is definitive treatment if mother life
is compromised. (Very high uncontrolled BP,
platelet <100, AST>150 iU/L
2. Can wait until term if well controlled and fetal
is not compromised.
3. If gestation >34W, then delivery after
stabilization is recommended
4. In this case, prolong delivery for 24 Hr to give
steroid injection for lung maturity
[RCOG Guideline No. 10(A) March 06]

Investigation
Repeat Dipstick testing within 6H
PE shows by urinary albumin
>300mg/24 hour@ >1g/l in 2 random
urine 6 hour a part.
1+ = 0.3 g/l, 2+ = 1 g/l and 3+ = 3 g/l.
24 Hour proteinuria to see severity of
PE. Severe PE >5000mg/24 hr.
BP should be checked every 15 minutes
until women are stable. Then,
Close monitoring of BP (at least
4Hourly) + reflex, clonus.
12

Long Case Examination for Phase III Medical Students


University Science Malaysia
Case: 19/M/F, G1P0 at 32W of pregnancy
diagnosed with pre eclampsia at 28W of
gestation.

1g/h for at least 24h after last seizure or delivery


Add 4 vials (10g) to 50cc of normal saline & run
at 5cc/h

Question
a) Signs and symptoms of impending
eclampsia
b) Magnesium sulphate

If further fits occur give a further


furth slow IV dose
of 2g & continue the maintenance infusion
Contraindications for Magnesium Sulfate:
Cardiac failure
Acute renal failure

Signs and symptoms of impending eclampsia


1) Headache
2) N & V
3) Visual Disturbances
4) Right upper quadrant pain
5) Progressively oedema (non dependant)
6) Frothy urine (proteinuria

Drug monitoring:
Clinical
1) Patellar reflex:
- After completion of loading dose
- Half hourly whilst on maintenance
maintenanc infusion
- use elbow reflex if epidural in situ
2) Respiratory rate: should be >16/min
3) Hourly urine output: should be >25ml/h
(urine output is critical as serum Mg level
depends on renal excretion)

Magnesium sulphate
Magnesium sulfate is superior to other AED
(phenytoin, diazepam).
Indications:
1) Eclampsia
2) Fulminating severe PE either:
a) Severe hypertension (MAP: >125
mmHg, SBP: >170 mmHg, DBP: >110
mmHg); OR
b) Hypertension with proteinuria (BP:
>180/90
mmHg,
proteinuria:
>0.3g/24h), AND one of the following:
i.
Clonus (>3 beats)
ii.
Severe persistent headache
iii.
Visual disturbance
iv.
Epigastric pain
v.
Platelet count <100 x 103/dL

4) Pulse Oximetry : must remain >90%


Serum Mg level should be checked when:
Oliguria (<25ml/h)
Respiratory rate <16/min
Pulse oximetry <90%
Continuing fit
Toxicity (therapeutic range: 2-4
2 mmol/l @ 4-8
mg/dl)
Loss of patella reflex
Weakness
Nausea
Feeling of warmth
5mmol/l
Flushing
Double vision
Slurred speech
Muscle paralysis
6-7 mmo/l
Respiratory arrest
Cardiac arrest
>12 mmol/l

Protocol for use of Magnesium Sulfate:


(5ml vial contain 2.5g MgSO4 ~0.5g/ml)
Loading Dose 4g Magnesium Sulfate
8ml (4g) + 12ml 0.9% saline IV over minimum
of 10 - 15 minutes

[Labor suite Management by Dr Adibah Ibrahim]

Maintenance Dose
13

Long Case Examination for Phase III Medical Students


University Science Malaysia
Case: 34/M/L, G2P1 C/C-High blood pressure
Dx- Essential hypertension.

Investigation
1) ECG
2) Urine dipstick test
3) Fasting Lipid profile
4) BUSE and creatinine,

Question: Hx and Pe only


Essential hypertension
-Primary elevation of blood pressure without
known causes which can be ameliorated only by
lifelong pharmacological therapy [Kumar&
Clark 6th edition]

Management
Non pharmacological
1. Lifestyle medication with light exercise.
2. Reduce the intake of salt and fat.

Risk factor
- Genetic
- Low birth weight
- Environmental factor
a) Obesity
b) Alcohol intake
c) Sodium intake
d) Stress
e) Smoking
- Humoral mechanism (insulin resistance)

Pharmacological
1. Stop ACE inhibitor and ARBs. Atenolol
can cause IUGR and Labetolol is
relatively contraindicated in Asthmatic
patient.
2. T. Methyldopa 250mg tds, max 3g/day
or
3. T. Labetolol 100 mg tds, max 300mg tds
or
4. Tab. Nifedipine 10 mg tds stat dose
** Do not give Methyl dopa together
with Nifedipine.
5. High calcium supplementation of 1.5
g/day to prevent PE
6. Avoid Combined vitamins C and E (in
the form of tocopherol from soybean) as
it may cause IUGR

Cardiac output rises in pregnancy, however there


is relative greater fall in peripheral resistance,
therefore BP in pregnant woman is usually low
than those not pregnant [Kumar& Clark 6th
edition]
Important history to be elicited
1) Risk factor to develop pre eclampsia

Others measurement
1. Routine ante natal check up.
2. Advise patient to come immediately to
hospital if develop signs and symptoms
of impending PE.
3. Urinary Dipstick to screen new onset of
proteinuria.
4. CTG and ultrasound to monitor fetal
well being.
5. Re assurance to the patient.
6. Can allow delivering via SVD unless
there is indication for C-Sec.

a. existing chronic medical disorders such


as obesity, hypertension, diabetes
mellitus, renal disease, connective tissue
disease and thrombophilia,
b. Previous history of preeclampsia or
eclampsia or IUGR or unexplained
stillbirth
c. Family history of preeclampsia or
eclampsia, and
d. Extremes of reproductive age (below 20
or above 40 years old)

14

Long Case Examination for Phase III Medical Students


University Science Malaysia
25 Years Old Malay lady, Nurse, G1P0 at date
+ 5/7 was admitted to wards because of
contraction pain and URTI. Patient also was
investigated for GDM because of excessive
weight gain during 21 week of pregnancy.

b) 2 hour post glucose load: 7-8


mmol/L
2) Level of blood glucose control: Blood
Sugar Profile (4-6 mmol/L) and Serum
HBA1c concentration (< 6.5%)

Questions
1) What Is GDM?
2) How do you diagnose GDM
3) Screening test for GDM
4) When to do MOGTT
5) Name the diabetogenic hormone in
pregnancy
6) What is normal weight gain in
pregnancy?

Screening test for GDM before performing the


MOGTT
a) Random blood sugar (> 11.1 mmol/L)
b) Urinary glucose level ( 1+ on more
than one occasion or 2+ on one
occasion)
c) Mini Glucose Tolerance Test (> 7.8
mmol/L)

Answer

When to do MOGTT
1) Candidates for MOGTT is offered for
this test at 16-18 weeks of pregnancy
2) If normal, then repeat at 26-28 weeks of
pregnancy. If it negative, then no need
to re-do it as HPL diabetogenic effect
starts to plateau even though its serum
level
continue
to
increase
proportionally.

What is GDM?
A syndrome of glucose intolerance appears
during pregnancy and usually disappears after
pregnancy is terminated. It affects 7% of all
pregnancy.
It is a metabolic disorder of multiple aetiology
characterized by chronic hyperglycemia with
disturbances of carbohydrate, fat and protein
metabolism resulting from defects in insulin
secretion, insulin action, or both.

Diabetogenic hormone in pregnancy


a) Human Chorionic Somatomammotropin
(HCS) or formerly known as Human
Placental Lactogen (HPL)
b) Estrogen (stimulate production of
prolactin)
c) Progesterone
d) Cortisol
Notes: In GDM, besides of anti-diabetogenic
hormone, there will be increased in insulin
degradation by placental enzymes

Previously, it is categorized into impaired


glucose tolerance test and GDM based on fasting
and 2 hour post glucose load in MOGTT.
However, current guidelines stated that GDM
includes impaired glucose tolerance test.
Diagnosing Diabetes Mellitus and the level of
blood sugar control
1) Diagnose: Based on MOGTT
Normal level of MOGTT is
a) Fasting: 5-6 mmol/L

Normal weight gain in pregnancy


1) First 5 months: 0.5 kg/months
2) Followed with: 0.5 kg/ week.
15

Long Case Examination for Phase III Medical Students


University Science Malaysia
Neonate
a) Congenital abnormalities
b) Shoulder dystocia, birth asphyxia & traumatic
birth
c) Hypoglycemia fetal islet cell hyperplasia
d) Jaundice
e)Respiratory
distress
syndrome

hyperinsulinaemia
diminished
surfactant
production
f) Hypocalcaemia and hypomagnesaemia

Case: 28/M/F, G3P2 at 28W P.O.A admitted in


view of uncontrolled blood sugar level.
Diagnosed as GDM at 26W P.O.A. Previous
pregnancy also complicated with GDM and
macrosomic baby requiring LSCS. Positive
family history of DM on maternal side.
Questions
a) Complication of GDM
b) Indication for MOGTT
c) Management to this patient

Indication for MOGTT


1) Significant glycosuria on 2 or more
occasions during pregnancy
2) Maternal obesity (i.e. maternal weight
>80 kg or BMI >27 at booking)
3) Family history of diabetes in first-degree
relatives
4) Previous big baby (weighing >4 kg)
5) Women >35 years old
6) Previous
unexplained
stillbirths,
recurrent abortions, birth defects
7) Previous history of gestational diabetes
8) Polyhydramnios in current pregnancy
9) Big baby in current pregnancy
10) Congenital abnormality

Complication of GDM
Maternal
a) Hypertension, incidence of preeclampsia (if a/w nephropathy)
b) incidence of infection UTI,
vulvovaginitis etc
c) Polyhydramnios
d) Pre-term labour
e) Coronary artery disease
f) Thromboembolic disease
g) Risk of caesarean delivery
Fetus
1. Early pregnancy
a) Spontaneous abortion
b) Congenital anomalies 40% of perinatal
death in diabetic pregnancies
c) Cardiac defects
d) Neural tube defects
e) Renal anomalies
f) Caudalregression synd (rare)

Management for this patient


My point of view: This patient was diagnosed as
GDM at 26W of pregnancy. Now is her 28W of
pregnancy and her blood sugar level is
uncontrolled. Obviously DM diet is not working.
Therefore, I see the role of giving insulin
injection to her.

2. Later pregnancy
a) Macrosomia
b) Polyhydramnios
c) IUGR (intrauterine growth restriction)
d) Unexplained intrauterine death. May be
secondary to:
Chronic hypoxia
Polycythemia
Lactic acidemia
Ketoacidosis

Therefore for this current admission, BSP should


be done after giving insulin injection to look for
the blood sugar level and further adjustment of
insulin dosage.
Pregnancy shouldnt be allowed beyond 38W
due to risk of unexplained IUD.

16

Long Case Examination for Phase III Medical Students


University Science Malaysia
Caesarean section
1) This is possibly a best option but this
will put patient in high risk category for
next pregnancy which is 2 caesarean
scars with no successful VBAC.
2) If patient wish to pregnant again, she
will require caesarean section for the
following pregnancy.

36 years old Malay lady, teacher, G4P3 at 37W


+ 6/7 was admitted to wards for further
management in view of
1) Establish DM for three years.
Previously on OHA but now changed to
insulin. However, blood sugar is
uncontrolled. Currently there is no
complication of DM develops.
2) Last pregnancy is by caesarian section
because of transverse lie.

Management for this patient


Antenatal
1) Fetal surveillance with ultrasound for
biophysical profile and CTG.
2) Blood sugar profile with adjustment of
insulin dosage.
3) Diabetic diet

Questions
1) Option of mode of delivery and pre
requisite for it.
2) Management for this patient.

Mode of delivery
In this patient, mode of delivery should be
balanced between benefit and risk. The decision
should always be discussed with the patient.

Intrapartum
1) Management based upon modes of
delivery either chooses induction of
labour with spontaneous vaginal
delivery or caesarean section.
2) Patient should be started on DKI
regimes (5% dextrose solution with 1
gram KCL) together with sliding scale
insulin infusion. If patient go for c-sec,
morning dose of insulin should be
omitted.
3) Presence of senior obstetrician to
standby in case any complication occur.
4) Pediatrician needs to be informed
regarding this case.

Spontaneous vaginal delivery with induction of


labour.
1) Should be done carefully if using
Prostin because of history of c-sec with
no successful VBAC. Dosage is 1.5 mg
for each cycle. Membrane sweeping
could be considered.
2) Need to elicit the lie of the fetus in
cephalic presentation.
3) Excludes macrosomic baby.

Post partum
1) Baby should be observed in NICU for
24 hours before discharged.
2) After the delivery, insulin can be stop
and patient may continue taking OHA.
3) Referral to internal medicine team for
further management
4) Advise for contraception.
5) Counseling on blood sugar control if
patient wish to get pregnant again

Answer
Patient with uncontrolled diabetes mellitus
should not be allowed to proceed with
pregnancy beyond 38 weeks of pregnancy.
Therefore, it is crucial to determine the correct
date of pregnancy to avoid pre term delivery.
Furthermore, fetus of diabetic mother is
associated with delay lung maturity.

17

Long Case Examination for Phase III Medical Students


University Science Malaysia
b) > 4,250 g = elective caesarean section

35 Years old Malay lady, G1P0 at 37W + 5/7


with gestational Diabetes Mellitus was admitted
for review for intrapartum management

Notes: Ultrasound is specific for determination of


estimated fetal weight but only with sensitivity of 6070% at term. There will be a + of 500 mg
discrepancy of estimated and real fetal weight.

Questions

Macrosomic baby of diabetic vs. non diabetic


mother

1) What should you elicit before allowing


patient to deliver by vagina delivery?
2) What is macrosomic baby?
3) Are there any differences between
macrosomic baby who is belonging to
diabetic mother and non diabetic
mother?
4) If this patient keen on SVD even though
the estimated fetal weight is 4 Kg and
the labour is complicated with shoulder
dystocia, what would be your
management?

Macrosomic baby of non diabetic mother is at


low risk for developing shoulder dystocia as
compared to baby of diabetic mother. This is due
to present of excessive fat tissue growth at
shoulder region in baby of diabetic mother. The
disproportionate excessive growth of the
shoulder will predispose them to the risk of
shoulder dystocia during SVD.
Steps in managing Shoulder dystocia

Answer

1) Call for help, inform senior obstetrician


and pediatric colleague
2) Experienced obstetrician should be
present during second stage of labour
3) Mc Roberts maneuvers (Flexion and
abduction of the maternal hips,
positioning the maternal thighs on her
abdomen)
4) If not successful, apply suprapubic
pressure together with Mc Roberts
(External suprapubic pressure is applied
in a downward and lateral direction to
push the posterior aspect of the anterior
shoulder towards the fetal chest )
5) If fail, proceed with Wood-Corkscrew
Maneuvers (The hand is placed behind
the posterior shoulder of the fetus. The
shoulder is rotated progressively 180 in
a corkscrew manner so that the impacted
anterior shoulder is released.
6) If still fail, then deliver the posterior arm
first.
7) If fail, do Zavanelli maneuvered (push
the baby back) and prep for emergency
C-sec

Before allowing diabetic mother deliver via


SVD, few thing needs to be excluded first.
1)
2)
3)
4)

The size of baby is not macrosomic


Cephalic presentation
Longitudinal lie
Not a candidate for Caesarean section
a) Major placenta previa
b) Footling or flexion breech
c) 2 previous c-sec scar without prior
normal delivery
d) Unstable lie
e) Any obstruction to descending of
fetus
(fibroid,
ovarian
cyst,
Cephalopelvic disproportion)
Macrosomic Baby

For undergraduate level, macrosomic is the


estimated weight of fetus > 4 kg. However, it is
further classified into categories
a) 4,000 - 4,250 g (discuss with patient
regarding mode of delivery)
18

Long Case Examination for Phase III Medical Students


University Science Malaysia
2) Observation of fetal condition through
serial ultrasound. Check for any
abnormality like spina bifida (associated
with hydrocephalus)
3) 30 minutes CTG monitoring for fetal
condition.
4) FBC and GSH for the mother
5) Blood sugar profile, Hba1c level of the
mother.
6) Check for any complication of diabetes
mellitus.

Case: DM with hydrocephalus baby


Question
a) H(x) and P(e)
b) Investigation and management
History
1) Regarding DM
a) Since when? Pre existing or during
this pregnancy
b) Any history of macrosomic baby,
Polyhydramnios or unexplained
IUD during previous pregnancy?
c) Are there family risk factor?
d) Is MOGTT done? (normally early
pregnancy and repeated at24-28w in
high risk group in which initial test
is negative)
e) Now on diabetic diet, OHA, or
insulin.
f) Ever being admitted due to DM
complication like hypoglycaemia,
diabetic foot.
g) Any complaint of DM complication
like heart disease, peripheral
vascular
disease,
diabetic
nephropathy, diabetic retinopathy.

Management
1) Prenatal
a) Pre term delivery is unlikely in this case;
therefore corticosteroid injection is not
needed.
b) Admit the patient at obstetric wards to
observe the blood sugar level. Starts
with diabetic diet. If fails, starts insulin.
c) Inform the pediatrician and neonatal
neurosurgeon regarding delivery of baby
and next intervention. (most likely
caesarian section at 38-39w to prevent
head entrapment)
d) Counseling to the patient regarding the
baby condition. Congenital abnormality
in DM is low. On next pregnancy should
take folic acid to reduce risk of
hydrocephalus.
e) Termination of pregnancy is against
medical ethics and Islamic law. Only
fetus which is dead in vitro or no chance
of living can be terminated.
2) Intrapartum
a) Prep for C-sec
3) Post natal
1) Check CBS of the baby and mother
2) Admit baby to the NICU for further
management.
3) Counsel mother to control diabetes and took
folic acid before next pregnancy

2) Regarding hydrocephalus
a) How did the patient know that?
Through US (usually diagnosed
after >24w)? Who confirmed it?
b) Did mother took/compliance to folic
acid?
c) Did
previous
baby
having
congenital anomaly?
d) The weight of the baby?
e) P(e) for unstable lie.
Investigation
1) Find the causes of hydrocephalus.
TORCHES?
Bleeding?
Edward
syndrome?

19

Long Case Examination for Phase III Medical Students


University Science Malaysia
In the term or post-term gestation,
oligohydramnios is frequently associated with
thick meconium (a/w Meconium Aspiration),
deep decelerations in the fetal heart rate, and
the dysmaturity syndrome. One team reported
a 13-fold increase in perinatal mortality rate
(to 57/1,000) when the sonogram showed
amniotic fluid volume to be marginal, and a 47fold increase (to 188/1,000) with severe
oligohydramnios.

Case: Oligohydramnios
Question
a) Complication of oligohydramnios
b) How to detect
c) Management
Definition
Reduce in AFI <5 based on ultrasound
[additional of vertical amniotic fluid pocket
depths volume in four quadrant.] Some specialist
may consider AFI <8 as oligohydramnios (AP
Dr Nik Hasliza).

In
62
cases
of
second-trimester
oligohydramnios, another team reported a 43%
perinatal mortality rate, with lethal pulmonary
hypoplasia complicating 33% of cases. If
amniotic fluid was essentially absent
("anhydramnios"), 88% had lethal outcomes,
compared with 11% of those with moderate fluid
reductions.

Amniotic fluid production


A) Production of amniotic fluid is from
1. Inward transfer of solute across the
amnion with water following passively in
early gestation.
2. Water transport across the highly
permeable skin of the fetus during the first
half of gestation (keratinization of skin at
22-25W)
3. Baby's urination (first starts at 8-11W
and is major source of production. it is
recycled when baby swallows it)
4. Secretion of large volumes of fluid each
day by the fetal lungs after second half of
gestation (2nd source)
B)
Increase amniotic fluid from 8-43W
gestation linearly until 32W (700-800 mLconstant until term)
-C) After 40W, declines at rate 8% per
week until 300ml at 42W

Diagnosis
- Via ultrasound
Management
Other Investigation
1) intrauterine instillation of dye to
diagnose PROM [confirm if the dye is
found in the vagina]-not practically done
2) Furosemide test to visualize fetal
bladder
 Both test not practically done
Others
1) Amnioinfusion of 200 ml Normal saline
(not practically done)

2) Maternal rehydration.(controversial)
3) frequent fetal biophysical testing and
appropriately timed delivery
4) Rule
out
fetal
structural
and
chromosomal anomalies
5) Earlier delivery in baby incompatible
with life.

Causes
1) PROM or PPROM
2) fetal urinary tract anatomy (renal and
ureter most common)
3) Uteroplacental insufficiency
4) Pulmonary hypoplasia

Notes: risk of fetal asphyxia and death is high in


IUGR

Complication

20

Long Case Examination for Phase III Medical Students


University Science Malaysia
35 years old Malay lady, G3P2 at 26W POA
was admitted for further management after she
persistently worried about her current
pregnancy because her belly was too big
compared to previous pregnancy

moderate (AFI 30.1-35) and severe (AFI >35)


[Naser Omar et al]
Causes of polyhydramnios
1) 60% is idiopathic
2) Maternal causes
a) Gestational diabetes mellitus
3) placental
abnormalities
(placental
abruption, placenta accreta)
4) Fetal factor
a) congenital anomalies ( anencephaly,
hydrocephalus,
spina
bifida,
tracheoesophageal fistula, duodenal
atresia, hydrops fetalis and many
more)
b) Multiple pregnancy
c) chromosomal abnormalities such as
Down's syndrome and Edwards
syndrome
5) Skeletal dysplasia and syndrome.
6) others like chorioangioma of the
placenta

Questions
1) What is polyhydramnios and how do
you grade them?
2) What is the causes of polyhydramnios
3) How do you manage this patient?
Answer

95th percentile
Mean value
5th percentile

Management to this patient


Source:http://emedicine.medscape.com/article/40485
6-overview

1) Reassure the mother


2) Excludes the causes of polyhydramnios
a) This patient should be offered to do
MOGTT
b) Ultrasound
examination
and
proceed to Doppler and full scan if
necessary
3) Assessment of fetal well being
a) Access while doing ultrasound +
CTG.
4) Treat the underlying causes
5) Treat the hydramnios
a) Mild & Moderate: Indomethacin or
sulindac
b) Severe: Amnioreduction
6) Corticosteroid if anticipating pre term
delivery.

Polyhydramnios
Polyhydramnios may be defined as an amniotic
fluid index above the 95th centile for gestational
age [Moore& Cayle].
Previously, it is defined when the deepest
vertical pool is more than 8 cm, but currently
based on measurement on 4 quadrant > 25.
(Based on ultrasound)
It complicates approximately 0.4-3.5 % of
pregnancies and it can be divided into three
groups: mild (amniotic fluid index 25-30),

21

Long Case Examination for Phase III Medical Students


University Science Malaysia
Tocolysis has also been advocated for the
management of intrapartum fetal distress,
impaired fetal growth, pre term labour and to
facilitate external cephalic version at term

Case: 32/M/F, G2P1, decreased fetal movement


Question
1) H(x) and P(e)
2) Management
of
decrease
fetal
movement
3) Use of tocolytic (function/type)
4) Fetal kick chart (indication and
component)

MgSO4
1. works
as
membrane
stabilizer,
competitive inhibition of Ca; therapeutic
at 4-7 mEq/L
2. SE: flushing, nausea, lethargy, pulm
edema
3. Toxicity: cardiac arrest (tx: calcium
gluconate), slurred speech, loss of
patellar reflex (@ 7 -10), resp problems
(@15-17),
flushed/warm (@9-12),
muscle paralysis (@15-17), hypotonia
(@10-12)

Reduce fetal movement? Baby goes through


normal sleep cycle. As long as baby moves
every couple of hour, then its fine.
History
Exclude Abruptio placenta
-Decreased fetal movement, abdominal pain,
bleeding after 22w
-shocks, tender uterus, fetal distress/absent fetal
heart sound

Nifedipine
1) calcium channel blocker: 10 mg q 6 h;
se: nausea and flushing

Exclude fetal distress


-Decreased/absent fetal movement, abnormal
fetal heart rate
- Thick meconium stained fluid

B2 agonist
1. ritodrine/ terbutaline
2. dec. uterine stimulation; may cause
DKA in hyperglycemia, pulm edema,
n/v, palpitations (avoid with h/o cardiac
disease or if vaginal bleeding) 0.25 mg
sq q 20-30 min x 3 then 5 mg q 4 po

Other history
- What did patient do? Working mother seems to
perceive less fetal movement.
- Any history of trauma?
- Elicit maternal medical illness

Indomethacin/prostaglandin synthesis inhibitor


1. 50 mg po/100 mg pr SE: premature
closure
of
PDA
in
an
hour,oligohydramnios

PE and investigation
1) Auscultation of fetal heart rate and
confirmation with ultrasound.
2) CTG monitoring for hour.
3) Umbilical artery Doppler ultrasound in
high risk cases.

Fetal kick chart


1) Screening by caregivers to alert them about
their fetal condition which might compromised.
This will aid early intervention to reduce
perinatal mortality.
2) Routine or done in women with increased risk
of complication in baby
3) Decision of management shouldnt be made
based on fetal kick chart.

Tocolysis
The administration of medications to stop
uterine contractions during premature labor

22

Long Case Examination for Phase III Medical Students


University Science Malaysia
about decrease in fetal movement as
compared with multi para.
2) Identification of maternal risk factor
which might contribute to perinatal
mortality.
- age,
smoking,
overweight/obesity,
previous stillbirth or neonatal death

Case: Reduce fetal movement


Question
a) Regarding traditional medicine, how to
advice patient
b) Line of thinking to get diagnosis
c) Management.

3) What actually the causes of reduce or


absent fetal movement?
a) Placenta Abruptio
b) Intra uterine growth restriction
c) Syndromic baby
d) Placenta insufficiency.
e) Mothers perception.

Traditional medicine
A doctor has no right to order patient to stop
taking traditional medicine. However, lack of
study and information between interaction of
traditional medicine and modern medicine may
cause few un-expected side effect.
Furthermore, few manufacturers being dishonest
by adding some hidden ingredient inside their
product which may cause serious side effect in
reaction to certain drugs. Therefore, as a doctor
we can advise patient to
1. Choose either taking only traditional or
modern medicine or not combining
them.
2. Suggest to them to stop traditional
medicine while pregnant because afraid
of unexpected side effect with
prescribed medicine.
3. Avoid herbal base traditional medicine.
4. Use alternative traditional medicine that
known scientifically not harmful like
honey.

4) Investigation to support diagnosis


Management
1) Take full history and elicit risk factor
that might compromise fetal condition.
2) Fetal
well
being
assessment
(recommended by NICE guideline)
CTG, Ultrasound.
3) Fetal kick chart (not recommended by
NICE and others as it will cause more
anxiety to the mother.) however, some
says it is better than doing nothing.
4) If CTG or ultrasound shows fetal
compromise, admit patient to the wards
and do serial monitoring of fetal
condition
5) Re assures the mother.
6) Patient can be safely discharge after
fetal monitoring shows normal result in
three consecutive days. Discharge
patient with
a) TCA at antenatal wards weekly or
twice weekly
b) To come again to ward if reduce
fetal movement
c) Instruction to use fetal kick chart.

Line of thinking to get the diagnosis


1) Is mother really paying full attention
about fetal movement
a) Fetal movement is rather perception
of woman. Busy mother tends to
feel less fetal movement.
b) Working in busy environment may
cause less perception of fetal
movement.
c) A woman which is first time
pregnant may become too anxious
about fetal condition and notice
23

Long Case Examination for Phase III Medical Students


University Science Malaysia
glucose tolerance, osteoporosis and
depression of fetal/maternal adrenals
2) Tocolytic
Nifedipine
- calcium channel blocker: 10 mg qds;
- se: nausea and flushing
B2 agonist
- ritodrine/ terbutaline
- dec. uterine stimulation; may cause
DKA in hyperglycemia, pulm edema,
n/v, palpitations (avoid with h/o cardiac
disease or if vaginal bleeding) 0.25 mg
sq q 20-30 min x 3 then 5 mg q 4 po

Case: Premature labour with PV Bleeding


Questions
a) 4 drugs in management of premature
labour
b) Drugs for candidiasis
c) Function, complication and monitoring
of the drugs
d) Doses of drugs
Definition of preterm
1) Onset of labour after the gestation of viability
i.e 24 weeks and before 37 completed weeks of
pregnancy.
2) The onset labour may be determined by
documented uterine contractions and rupture
membranes or documented cervical change with
an estimated length of less than 1 cm and/or
cervical dilatation of more than 2 cm.

3) Antibiotic therapy
- For women at risk of preterm delivery
because of PPROM, prophylactic
antibiotics delay delivery and reduce
maternal
and
neonatal
infective
morbidity.
- Not recommended in risk of preterm but
with intact membranes
- Erythromycin 500mg qds plus coamoxyclav (Augmentin) 375mg tds for
7 days OR clindamycin 150mg qds for 7
days.

Types
a) Threatened (uterine contraction without
cervical changes)
b) Actual/establish (uterine contraction+ cervical
changes)
Additional: occurs in around 7% of all
pregnancies and is a major cause of infant
mortality and morbidity. [Scottish guidelines]
Survival rate: 23 w 0-8%
25w 50-60%
26-28w 85%

Drug for candidiasis in pregnancy

24w 15-20%
29w 90%

Imidazoles are best but pregnant women may


need longer (7 not 4 day) courses. Thrush is a
common vaginal infection in pregnancy causing
itching and soreness. There is no evidence that
this yeast infection harms the baby. Antifungal
creams are effective. Imidazoles (such as
clotrimazole) are more effective than older
treatments such as nystatin and hydrargaphen.
Longer courses (7 days) cured more than 90% of
women whereas standard (4 day) courses only
cured about half the cases. [Cochrane Database
of Systematic Reviews, Issue 4, 2009]

Drugs in management of premature labour


1) Corticosteroid therapy
- Betamethasone, 12mg, IM, 24 hours
apart.
- In USM, Dexamethasone, 12 MG, IM,
12 hours apart.
- Function is to increase lung maturity.
Usage of corticosteroid below 24w is no
beneficial since pneumocyte not develop
yet. Also not recommended >34W.
- Possible long-term effects on cognitive
or neurological development, impaired
24

Long Case Examination for Phase III Medical Students


University Science Malaysia
4) Re assure the patient
5) Give IM Dexamethasone 12mg bds, 12 hours
apart.
6) Keep patient nil by mouth and anticipate for
caesarian section.
7) Hydrate patient adequately with 2 pints NS
and 3 pints D5%
8) Take blood for investigation including FBC,
GSH.
9) Urine dipstick (nitrogen, albumin= indicate
UTI) and Urine FEME.
10) Allocate possible causes of premature
contraction.
11) Monitor 4 hourly BP, 20 minutes CTG
12) Inform Pediatrician regarding patient's
condition and keep in view to book for
ventilator.
13) Monitor the labour progression by labour
progression chart.
14) Ultrasound examination for fetal well being.
15) Administer tocolytic for example Nifedipine
5mg (some specialist give 10 mg)
16) Observe patient for one day. if contraction
subside, discharge patient to ante natal wards for
further observation.
17) If contraction subsides for two consecutive
days in ante natal wards, then patient can be
discharged

Case: 33 years old, G3P2 present at 24W+5D


POA with premature contraction. No history of
UTI, Vaginal discharge, trauma.
Question
A) How
to
differentiate
premature
contraction and true labour contraction
B) Management of this patient
C) How to discharge this patient
Premature contraction
Uterine contraction after the gestation of
viability. i.e 24W and before 37 completed
weeks of pregnancy. It could progress to
premature labour.
It is called threatened pre term labour if
contraction is not associated with cervical
dilatation.
If it is associated with cervical dilatation, hence
it is termed as Establish Pre term labour.
Characteristic of a true labour
1) At term
2) Come at regular interval i.e once in one
hour and finally can goes to once in five
minutes near labour.
3) The timing of each contraction is last
about 30-70 seconds
4) The intensity of pain increase by time.
Pain is at the back due to referred pain
of cervix.
5) The pain does not relief by walking or
changing in posture.
6) Presence of show and liquor.

Discharging the patient


1) After no contraction within 2 days in ante
natal wards
2) Ensure that patient already took
dexamethasone.
3) CTG reactive.
4) Follow up at ante natal clinic within 2 weeks.
5) Fetal Kick Chart (some protocol say it is not
indicated)

Management of this patient


1) Obtain full history and perform relevant
physical examination.
2) Admit patient to premature room in labour
room.
3) Inform the case to MO in charged
25

Long Case Examination for Phase III Medical Students


University Science Malaysia
fetal, placenta abnormalities (PE, Anruptio
placenta, PP), multiple pregnancy, Medical
condition like Hypertension, DM and anemia,
aneuploidy and fetal anomalies, increase
maternal mortality. [The older obstetric patient,
Current Obstetrics & Gynecology (2005) 15,
4653]

Case: 42/M/F G11P7+3A at 36/52 with history


of 5 premature deliveries, electively admitted for
removal of cervical cerclage
Questions
a) Risk of grandmultiparae
b) Risk of pregnancy at old age
c) How to prevent PPH in this patient
d) Indications of cervical cerclage
e) When to do and remove cervical
cerclage.

Cervical cerclage
Cervical cerclage is a procedure in which sutures
are inserted around the cervix in women
suspected to have cervical weakness. This is
thought to prevent cervical dilatation and
membrane exposure, thus helping the uterus to
retain the pregnancy in women who are prone to
miscarrying, mostly in the mid-trimester.
[Cervical cerclage, Current Obstetrics &
Gynaecology (2006) 16, 306308]

Grandmultiparae
Definition: a woman who has had five or more
pregnancies resulting in viable fetuses. Great
grand multipara if > 10
The results showed high in incidence of anemia
(80%). Cesarean section (38% vs 35%),
inversion of uterus (0.2% vs nil) and rupture of
uterus (0.2% vs nil), hypertension and PIH
superimposed on chronic hypertension (12.525% vs 8-14%). The incidence of postpartum
hemorrhage, abruptio placentae, preterm labour,
obstructed labour, puerperal sepsis and wound
infection was also high in grand multi parous
group. There were 9 maternal deaths out of 1000
cases of study group as compared to 4 deaths in
control group. Similarly the perinatal mortality
rate was 180/1000 births as compared to
150/1000 in para 2-4. [Grand Multiparity: Still
an Obstetric Risk Factor,Khadija H Asaf. Pak J
Obstet Gynaecol May 1997;10(1,2):24-8]

Cervical cerclage can be classified as an elective


procedure (based on previous history and/or
investigation), a selective procedure (based on
evidence obtained by ultrasound examination
that shows shortening of the cervix) or an
emergency procedure (when the cervix is dilated
with the membranes seen or bulging via the
cervical os).
An elective procedure is performed around 12
14 weeks gestation (Dr Amir HUSM-16W)
after confirming fetal viability. The TAC is
performed around the same time. Emergency
cerclage is performed when the cervix is noted
to be dilating.[Cervical cerclage, Current
Obstetrics & Gynaecology (2006) 16, 306308]

Pregnancy at old age


an age over 35 years for the elderly
primigravida(FIGO, 1958)Improvements in
womens general health have led to this term
tending to be reserved for pregnancies in women
at or above 40 years of age.[Current Obstetrics
& Gynaecology (2005) 15, 4653]

It is removed when term is achieved or


premature contraction (Dr Amir HUSM)

Risk of pregnancy at old age: Miscarriage,


ectopic pregnancy, chromosomal disorder in
26

Long Case Examination for Phase III Medical Students


University Science Malaysia
For PIH
1) For baby(similar to post EDD)
2) For mother
a) Hourly BP monitoring
b) PE Profile (platelet count, uric acid,
serum creatinine level, Liver
enzymes- AST, Urine albumin)

Case: Post EDD + PIH (exact case is unknown


so discussion is random)
Question
a) Differential diagnosis
b) Investigation
c) Management

Management for Post date and post Term


Based on scientific evidence
1) Women with post term gestations who
have unfavorable cervices can either
undergo labor induction or be managed
expectantly.
2) Prostaglandin can be used in post term
pregnancies to promote cervical
ripening and induce labor.
3) Delivery should be effected if there is
evidence of fetal compromise or
oligohydramnios.

Term
Period of gestation 37 to 42 week
Post date
Post date is a term to describe any pregnancy
that goes beyond expected date of delivery (40
W) but does not exceed term.
Current practice in HUSM and KKM is to avoid
the delivery of post term baby due to the
increase perinatal morbidity and mortality
associated with post term. Therefore, induction
of labour should be initiated if pregnancy goes
beyond the post date.

Based on expert experiences


1) Antenatal surveillance for post term
pregnancies between 41 weeks (287
days; EDD +7 days) and 42 weeks (294
days; EDD +14 days) of gestation
because of evidence that perinatal
morbidity and mortality increase as
gestational age advances.

Approach to this patient


1) Absolute treatment for PIH is the
termination of pregnancy.
2) Post EDD itself is an indication for
induction of labour to prevent post term
delivery.
3) However, the exact POG needs to be
established to avoid delivery of pre term
baby.

1. Twice-weekly testing with some


evaluation of amniotic fluid volume
beginning at 41 weeks of gestation. A
nonstress test and amniotic fluid volume
assessment (a modified BPP) should be
adequate.

Investigation
For Post EDD
1) Biophysical profile of the baby
Fetal tone
Movement of the body or limbs
breathing movement
Amniotic fluid volume
Heart rate (CTG) analysis.
2) Doppler ultrasound

Drug commonly use in PIH


a) Methyl dopa
b) Labetolol
c) Nifedipine
d) Magnesium Sulphate (pre eclampsia)

27

Long Case Examination for Phase III Medical Students


University Science Malaysia
In this patient, we however need to go through a
broad perspective before deciding the
management for this patient. Patients problem
is
a) Post date
b) 1 previous c-sec with successful 4
VBAC (low risk patient)
c) Grand multi para (deliver > 5 times)

31 years old Malay lady, G6P5 at date + 9/7


with 1 previous scar for transverse lie and 4
VBAC
1) What is post term?
2) What is the complication of the post
term?
3) In this patient, how will you manage her
and give reasons.

Therefore, risk and benefit on method of


delivery should be considered.

Post term
A pregnancy that has extended to or beyond 42
weeks of gestation (294 days, or estimated date
of delivery [EDD] +14 days) [ACOG
guidelines]

Roughly, this
management.

is

the

overview

of

the

1) Antenatal surveillance at 42 to 42 week


including at least non stress test and
assessment of amniotic fluid volume.
2) Estimating the fetal weight.
3) Considering the induction of labour
a) Prostin and oxytocin is not a good
choice.
b) May consider membrane sweeping
c) After excludes macrosomic baby,
breech presentation and severe
oligohydramnios.
4) If we considering Caesarian section
a) Indicated if it is a macrosomic baby
b) But this will put mother on higher
risk on next pregnancy because of
two c-sec scar and grand Multipara.
Risk of uterine atony and rupture is
high
c) Advise on bi tubal ligation for the
mother.
d) Indicated if breech presentation.
e) Indicated in severe oligohydramnios
5) Present of senior MO and pediatrician
during delivery.

Complication of the post term


1) To the baby
a) Uteroplacental insufficiency
b) Oligohydramnios
causing
cord
compression syndrome
c) Meconium aspiration syndrome
d) Intrauterine infection
e) Macrosomia and complication related to
it
f) Fetal dysmaturity syndrome
g) Increased
risk
for
neonatal
encephalopathy
2) To mother
a) Severe perineal injury if delivering big
baby
b) Increase rate for caesarian section
c) endometritis, thromboembolic disease,
hemorrhage.
d) Psychological (anxiety and frustration)
for carrying the baby longer than
expected.
Management for this patient
Women with an uncomplicated pregnancy who
reach 41 to 42 weeks gestation should be
offered elective delivery [SOGC]
28

Long Case Examination for Phase III Medical Students


University Science Malaysia
A healthy 25-year-old nulliparous woman has
an uncomplicated pregnancy at 42 weeks.
Induction of labour is fully discussed, suggested
and declined. Evaluate the tests that may be
arranged to monitor fetal health until labour
begins. [Journal review]

such events. Contraction stress tests assessing


fetal heart responses to oxytocic-induced
contractions have largely gone out of use as the
high false-positive rate has led to a high level of
intervention. They also take longer and are more
complicated to conduct than NSTs.

Where the advice over induction is unacceptable


to the patient, various measures may be used to
review fetal health. Several tests are described
for this situation, but the problem remains that
no test or group of tests has been shown to
improve the perinatal outcome.

The assessment of liquor volume that may be


related to placental function and fetal health has
become an accepted part of surveillance of
women such as this. A measurement of the
amniotic fluid index of less than 5 cm or of the
maximum vertical pocket depth of less than 2
cm (various other levels are quoted) suggest
fetal compromise and lead to a recommendation
for delivery. The biophysical profile combines
an ultrasound assessment of fetal movements
and tone, breathing movements and amniotic
fluid volume. This combination would seem to
be the most appropriate, but a recent RCT
showed no advantage over CTG with amniotic
depth measurement. Doppler studies of
umbilical artery velocimetry have not been
shown to be of benefit in predicting outcome.

Fetal movement charts have long been used in


late pregnancy as a form of fetal monitoring.
They require the woman to note the time taken
for 10 fetal movements. A large randomisedcontrolled trial (RCT), however, demonstrated
no reduction in fetal mortality compared with
the control group, and the use of these charts
may increase anxiety without a beneficial
effect. Various forms of fetal acoustic
stimulation test have been developed using a
transabdominal sound source and assessing the
fetal reaction in terms of cardiotocograph (CTG)
changes and fetal movements. Although such
tests have the advantage of taking less time than
other tests, there is no evidence for an improved
perinatal outcome.

There are clearly considerable limitations in the


value of all these tests in detecting fetal
compromise and enabling rescue, but NICE
recommends
twice-weekly
CTG
and
measurement of amniotic pool depth, and this
will remain the advice until further advice based
on RCTs is available. The advice and its reasons
must be discussed with this woman, and a
sensitive approach is most likely to lead to a
compromise, although there remains a
possibility that the woman will wish for no tests
and will await natures decision.

The standard non-stress test (NST) using a


CTG relies on the observation of two or more
episodes of fetal cardio acceleration of 15 beats
or more occurring within 20 minutes of the onset
of the test. It is generally thought, and advised
by the National Institute for Health and Clinical
Excellence (NICE), that twiceweekly tests
should be performed, although the optimum
scheme is not known. Furthermore, the NST
used alone has a low sensitivity. The most
common cause of perinatal death in such cases is
meconium aspiration due to an acute asphyxial
event, and the NST is not adequate to preclude

Simon G. Crocker Department of Obstetrics &


Gynaecology Norfolk & Norwich University
Hospital, Colney Lane, Norwich NR4 7UY, UK
[OBSTETRICS,
GYNAECOLOGY
AND
REPRODUCTIVE
MEDICINE
17:1,2007
Published by Elsevier Ltd.]
29

Long Case Examination for Phase III Medical Students


University Science Malaysia
Case: PPROM

1) Clear watery and alkaline per vaginal


discharge. (pH 7.1-7.3 compared with
vaginal pH 4.5-6.0)
2) Arborization
(ferning)
under
microscopic visualization
3) Oligohydramnios
[ACOG Practice Bulletin, VOL. 109, NO. 4,
APRIL 2007]
Symptoms of chorioamnionitis: High grade
fever, maternal and fetal tachycardia, tender
uterus.

Question:
a) Symptoms of fever
b) Positive findings in PPROM
c) Ix and Mx
d) Causes of unstable lie
PPROM
Membrane rupture that occurs before 37 weeks
of gestation is referred to as preterm PROM
Intraamniotic infection has been shown to be
commonly associated with preterm PROM,
especially if preterm PROM occurs at earlier
gestational ages. In addition, factors such as low
socioeconomic status, second- and thirdtrimester bleeding, low body mass index less
than 19.8, nutritional deficiencies of copper and
ascorbic acid, connective tissue disorders (eg,
EhlersDanlos syndrome), maternal cigarette
smoking, cervical conization or cerclage,
pulmonary disease in pregnancy, uterine
overdistention, and amniocentesis have been
linked to the occurrence of preterm PROM

Investigation and management


**Based on period of gestation but basically
1. Determination of gestational age, fetal
presentation, and well-being
2. Expeditious delivery in patient with
evident intrauterine infection, abruptio
placenta, or evidence of fetal
compromise
3. swabs for diagnosis of Chlamydia
trachomatis and Neisseria gonorrhoeae
if immediate delivery not indicated
4. Group B antibiotic prophylaxis
5. CTG monitoring for umbilical cord
compression or asymptomatic uterine
contraction.

The risk of recurrence for preterm PROM is


between 16% and 32%.
Fever
Fever is considered as temp above 100.40 (380C)
but feverish sensation may occur when body
temp above 98.60 (370C)

Causes of unstable lie


2. Prevention of head descending
a) Cephalopelvic disproportion
b) Fibroid
c) Ovarian cyst
d) Placenta previa
e) Uterine surgery
f) Multiple gestation
g) Fetal abnormality (anencephaly)
h) Fetal neuromuscular disorder

Symptoms
1) Patient complaints of body become hot
and sweating (increase temperature and
diaphoresis)
2) Can also a/w with tachycardia, altered
consciousness, chills & rigor, headache,
muscle and joint pain.

3. Condition that permit free


movement
a) Polyhydramnios (AFI>8)
b) Uterine laxation

Positive findings in PPROM

30

fetal

Long Case Examination for Phase III Medical Students


University Science Malaysia
25 years old Malay lady, G3P2 at 40 weeks +
2/7 of pregnancy presented to you because of
gushing of clear fluid from vagina for 1 day
duration. However, there is no contraction pain

1)
2)

Questions
1) Differential diagnosis and further
history to support your diagnosis
2) What complication that should concern
you that may occur to the mother and
baby.
3) How do you manage this patient

3)
4)
5)

6)
7)

Differential diagnosis
1) Pre labour rupture of membrane
- Establish that the fluid is truly amniotic
fluid and not urine.
- Contraction pain
- Liquor color

8)

2) In labour
- Contraction pain that become shorten in
intervals
3)
-

Notes:
Student must be able to differentiate between the
PPROM and PROM.

Bacterial vaginosis
Foul smelling discharge
Itchiness of vagina
Yellow or cream color discharge
Any systemic sign for infection.

PPROM is premature rupture of membrane. i.e;


membrane rupture during the period of fetal
viability (>24 W) but not reach term yet.

Complication to look for


1) Chorioamnionitis
Notes: chorioamnionitis must be
anticipated in patient presented with
PPROM or PROM as it can cause death
to the mother and fetus.
Beware sign and symptoms
chorioamnionitis
a) Fever
b) Maternal tachycardia
c) Tender uterus
d) Fetal tachycardia

Management
Close observation of vital sign
Review patient regularly especially
palpation of uterus.
a) Elicit uterine tenderness
b) Lie and presentation of the fetus
c) Estimated fetal weight.
Assessment of fetal well being (CTG
and ultrasound )
Rehydrate the patient
IV ampicillin as prophylaxis against
GBS (rate of infection rise after 12 hour
rupture of membrane)
Corticosteroid is not indicated.
Notify the pediatric team regarding the
possibility to admit the baby because of
infection.
This patient should not be discharged
and induction of labour should be
discussed with patient if not deliver after
>24 hour. Usually 90% of patient with
PROM will deliver within 24 hour.

Meanwhile, Prelabour rupture of membrane,


PROM is the ruptured membrane before labour.
The patient already at term.
In PPROM, the focus is 1) to prolong the
pregnancy so that the chances for fetus to
survive remain high and 2) to prepare the baby
for possibility to be delivered prematurely.

of

In PROM, a focus should be stress on possible


infection to the mother and also to the fetus.
Lung complication to the fetus is unlikely as
lung maturation already completed.
31

Long Case Examination for Phase III Medical Students


University Science Malaysia
Case: Placenta Previa

Investigation to order
1) Transvaginal or Abdominal US
2) FBC, GSH, Rh compatibility
3) CTG

Question
a) Management
b) Other name for low transverse scar
c) Types of placenta previa
d) Investigation to order
e) Complication
f) How to differentiate between placenta
previa and abruptio.

Management
1) Admission to ward for PP Major.
2) Bed rest and serial US as placenta
migration can occur.
3) Transfuse blood if needed i.e
symptomatic or near delivery, (target Hb
at delivery is at least 8) + haematinic.
4) Tocolysis and corticosteroid if prem
delivery is anticipated
5) Avoid Sexual intercourse
6) PP Minor can be allowed for SVD
7) Counseling to the patient.

Definition:
Placenta implanted in the lower segment of the
uterus, presenting ahead of the leading pole of
the fetus. It occurs in 2.8/1000 singleton
pregnancies and 3.9/1000 twin pregnancies
[MARCH JOGC MARS 2007]
Grade.
I

Placenta
Lateral
encroaches on the
lower uterine
segment but does
not reach the
cervical os.

II

Placenta reaches marginal


the margin of the
cervical os but
does not cover it.

III

Placenta partially
covers the os.

IV

Placenta is
symmetrically
implanted in the
lower uterine
segment

Complete

Complication:
Antepartum
-APH (3rd trimester) and Cx of blood transfusion
-Unstable lie
- Perinatal death
- Pre term labour
Thromboembolism d/t prolongs bed rest.
-Thromboembolism
- Placenta abruption

Minor

major if
posterior
located

Intrapartum
- Excessive bleeding during SVD
- C- section and its complication (major PP)
- Hysterectomy
Post partum
-DIC
- Intra uterine adhesion
-Recurrent PP
- Placenta accrete

Major

PP VS Placenta abruptio
Association with pain: PP is painless
US: abruptio shows Retro placental blood clot

Other name for low transverse scar


Pfannenstiel (traditional) scar, Joel--Cohen scar.

32

Long Case Examination for Phase III Medical Students


University Science Malaysia
21 Years Old Malay Lady, G1P0 at 36/52 + 4/7
was admitted since 30 Week POA after history
of sexual intercourse and strenuous activity.
Currently, there are no more bleedings.
Ultrasound reveals placenta previa type III

1)
2)

3)

Questions
1) How to differentiate PP type III and PP
type IV on ultrasound
2) What is the risk factor for placenta
previa
3) Can this patient be discharged?
4) Management for this patient

4)
5)
6)

How to differentiate PP type III and PP type IV


on ultrasound
PP type III- Placenta partially covers the os.
PP type IV- Placenta is symmetrically implanted
in the lower uterine segment

Risk factor for placenta previa


Multiple gestation
Previous caesarian section
Uterine structure anomaly
Assisted conception
Dilation & Curettage

7)

Management for this patient


FBC to access the level of hemoglobin
of this patient
GSH as patient may lose lot of bloods
during delivery. GXM 2 unit should be
prepared before delivery.
Tocolysis as bleeding in PP patient may
also due to contraction of uterus (not the
lower segment)
Assessment of fetal well being
Complete bed rest and avoid any
excessive activity.
Discuss with patient regarding caesarian
section as mode of delivery.
Prior to delivery, all women with
placenta praevia and their partners
should have had antenatal discussions
regarding
delivery,
hemorrhage,
possible blood transfusion and major
surgical
interventions,
such
as
hysterectomy, and any objections or
queries dealt with effectively.[RCOG]
To prepare the patient for caesarian
section.

Notes:
1) Trans vaginal ultrasound is safe in the
presence of placenta praevia and is more
accurate
than
trans
abdominal
ultrasound in locating the placenta
[RCOG Guideline No. 27
2) Placenta migration occurs during second
and third trimester except in posteriorly
located PP and present of C-sec scar.
3) A scan should be performed at 32 weeks
in case suspected PP major and 90% of
the patient who is diagnosed with PP
major will remains so.
4) Elective caesarean section should be
deferred to 38 weeks to minimize
neonatal morbidity.

Can this patient be discharged?


No,
Women with major placenta praevia who have
previously bled should be admitted and managed
as in patients from 34 weeks of gestation.
[RCOG]
All women at risk of major ante partum
hemorrhage should be encouraged to remain
close to the hospital of confinement for the
duration of the third trimester of pregnancy
[Royal Australian and New Zealand College of
Obstetricians and Gynecologists]
Women with placenta praevia who have bled
tend to deliver earlier
33

Long Case Examination for Phase III Medical Students


University Science Malaysia
Case: 42/M/F G9P8 with unstable lie

Management
1) Admit patient to antenatal wards
a) Daily observation for fetal lie
b) Provide active management to
correct lie
c) Provide
immediate
clinical
assistance upon membrane rupture
2) Exclude factors contributing to unstable
lie
3) Expectant vs. Emergent management

a) Causes of unstable lie


b) Management
c) Complication

Unstable lie
1. Fetal lie and presentation repeatedly
change at beyond 36/52 of gestation.
2. by 36W, fetal movement is limited, fetal
should present as cephalic)
3. Incident at 26/32 is 40%, at 30/52 is
20% & at term is 3%

Expectant
A) Daily observation for fetal lie
B) Discharge if longitudinal lie for 3
consecutive days
C) Review patient in a week time
D) Wait for spontaneous labour

Causes of unstable lie


4. Prevention of head descending
i) Cephalopelvic disproportion
j) Fibroid
k) Ovarian cyst
l) Placenta previa
m) Uterine surgery
n) Multiple gestation
o) Fetal abnormality (anencephaly)
p) Fetal neuromuscular disorder

Active management
A) Caeserean section
B) ECV
C) Stabilizing induction of labour

Complication
5. Condition that permit free
movement
c) Polyhydramnios (AFI>8)
d) Uterine laxation

fetal
1) Cord prolapsed leading to fetal hypoxia/
fetal death.
2) Compound presentation
3) Uterine rupture

History
a) Make sure that the date is correct
cause unstable lie is physiological
<36/52.
b) Find any risk factor associated with
unstable lie.
c) Elicit any problem during pregnancy

34

Long Case Examination for Phase III Medical Students


University Science Malaysia
Case: Unstable lie
Option for this patient
A) Passive management by observation
in hope that the lie will return to
normal position during term.
B) Caeserean section
C) ECV [Relative contraindication]
Results vary from 30% up to 80% in
different series. Race, parity, uterine
tone, liquor volume, engagement of
the breech and whether the head is
palpable, and the use of tocolysis,
all affect the success rate.
[Greentop]
D) Stabilizing induction of labour

Question:
a) Physical examination (abdomen)
b) Level of exposure
c) Clinical evidence of head and buttock
d) Management for this patient
e) Option for this patient
f) What is unstable lie
g) Causes of unstable lie
Unstable lie
1. Fetal lie and presentation repeatedly
change at beyond 36/52 of gestation.
2. by 36W, fetal movement is limited, fetal
should present as cephalic)
3. Incident at 26/32 is 40%, at 30/52 is
20% & at term is 3%

Notes: B-D is active management.


Complication

Clinical evidence of head and buttock


Head: Hard, round and ballotable
Buttock: Soft, broad and not ballotable.

1) Cord prolapsed leading to fetal hypoxia/


fetal death.
2) Compound presentation
3) Uterine rupture

Management
Causes of unstable lie
Prevention of head descending
a) Cephalopelvic disproportion
b) Fibroid
c) Ovarian cyst
d) Placenta previa
e) Uterine surgery
f) Multiple gestation
g) Fetal abnormality (anencephaly)
h) Fetal neuromuscular disorder

1) Admit patient to antenatal wards


a) Daily observation for fetal lie
b) Provide active management to
correct lie
c) Provide
immediate
clinical
assistance upon membrane rupture
2) Exclude factors contributing to unstable
lie
3) Expectant vs Emergent management
Expectant
A) Daily observation for fetal lie
B) Discharge if longitudinal lie for 3
days
C) Review patient in a week time
D) Wait for spontaneous labour

Condition that permit free


movement
a) Polyhydramnios (AFI>8)
b) Uterine laxation

35

fetal

Long Case Examination for Phase III Medical Students


University Science Malaysia
Case: Breech
Mode and timing of delivery
Questions
a) Causes and complication of breech
b) Management, mode of delivery and time
of delivery for breech.
c) ATT- Type of immune
Breech
It is the most common type of malpresentation.
Presentation of the fetal buttocks and feet in
labour

<28 weeks, weight <1


kg

SVD

28-32 weeks, weight


1.0 - 1.5 kg

LSCS

32-37 weeks Weight


1.5 2.5 kg

Depend on case
1) Assisted
breech
delivery
for
Extended and
flexed
2) LSCS
for
footling
breech
Preferably Caesarean
section

Incidence: 26W 40%, 30W 20%, term 3%


Type: Extended, Flexed, Footling

1)
2)
3)
4)
5)
6)
7)
8)
9)

Causes
Multiparous woman with lax uterus and
abdomen
Prematurity
Fetal structural anomalies; anencephaly,
hydrocephalus
Uterine anomalies; uterus bicornu,
fibroids
Multiple gestation; twins
Hydramnios; oligo or poly
Placenta previa
Contracted maternal pelvis
Pelvic tumours

> 37 weeks

ATT
Tetanus vaccine is an inactivated toxin (toxoid)
made by growing the bacteria in a liquid
medium and purifying and inactivating the toxin.
Type II Immune response
It is administer once the quickening felt and can
be repeated 2-3 months after first injection in
primid women as a booster injection. (Usually
5th and 7th months of pregnancy)

Complication
Notes: It is different from Anti tetanus human
immunoglobulin
which
are
preparation
containing IgG immunoglobulin derived from
plasma of donors sensitized to tetanus toxoid. It
acts by The IgG antibodies acts to neutralize the
free circulating exotoxisn of clostridium tetani
and prevent its fixation in tissue and its
consequences.

1) PROM
2) Cord prolapsed [common in footling
presentation and lesser in flexed breech
presentation]
3) Difficulty in delivering the shoulder
4) Difficulty in delivering the head[ may
lead to intracranial bleeding d/t tear of
tentorium or delay delivery of head can
cause prolonged compression of cord
and asphyxia]
5) Birth trauma such as fracture, viscera
damage, Erb Duchenne paralysis,
dislocation of hip joint.
36

Long Case Examination for Phase III Medical Students


University Science Malaysia
35 years old Malay lady, G5P5 (1 pair twin) at
26/52 of pregnancy was admitted to wards
because of premature contraction and twin
pregnancy.

b) Number of fertilized egg (zygosity;


mono,di)
c) Number of placenta (chorionicity)
d) Number of amniotic cavity (amnionicity)

Questions
1) What history that could give you idea
that you are dealing with cases of
multiple pregnancy?
2) How do you diagnosed multiple
pregnancy through physical examination
3) How do you classify multiple pregnancy
4) Complication of multiple pregnancy

Complication of multiple pregnancies


To the mother
1) Hyperemesis gravidarum and in fact all
physiological
response
towards
pregnancy will be exaggerated.
2) Exacerbation of chronic illness
3) Severe anemia in pregnancy
4) DIC secondary to fetal death.
5) Miscarriage
6) Preterm labour.
7) Polyhydramnios
8) Pre eclampsia.
9) Placenta abruptio
10) Post partum hemorrhage.
11) Higher risk for developing GDM

Answer
Notes: Account for only 3% of all live births,
they responsible of a disproportionate share of
perinatal morbidity and mortality
History
- Accelerated weight gain
- Hyperemesis gravidarum
- Sensation of moving of more than one
fetus
- Infertility treatment by ovulationinducing agents or gamete/zygote
transfer
- family history of dizygotic twins

Others: acute
embolism

fatty

liver,

pulmonary

To the fetus
1) IUD of one fetus
2) IUGR
3) Fetal abnormality
4) Pre term delivery
5) Low birth weight
6) Acute respiratory distress syndrome.
7) Congenital
abnormality
(mental
retardation, Siamese twin, cerebral
palsy)

Notes: certain race like Africa has higher risk


factor to have multiple pregnancies.

Physical examination
- Presence of more than 2 poles (need to
excludes fibroid)
- Usually, the abdomen size is bigger than
corresponds date
- Polyhydramnios
- Presence of two or more fetal heart
sounds on pinnard auscultation.

Others: twin-to-twin transfusion syndrome,


Twin reversed arterial perfusion (TRAP)/
acardiac twinning

Classification of multiple pregnancies


a) Number of fetus (twin, triplet, quadruplets)

37

Long Case Examination for Phase III Medical Students


University Science Malaysia
29 Years old Malay lady, G1P0, twin pregnancy
at 24/52 was admitted because of frothy color
urine and headache.

Management to this patient


1) Evaluation on severity of the pre
eclampsia
- Close monitoring of blood pressure (15
minutes interval until BP stable)
- Repeat Dipstick testing within 6H
- 24 hour urinary protein
- PE Profile (platelet count, uric acid
level, sr Creatinine level, liver enxyme)
- Clotting study if platelet < 100 x 106/l
2) Management of hypertension

Questions
1) What is your provisional diagnosis and
justify your answer?
2) What is twin to twin transfusion
syndrome?
3) Factors contributes to preterm delivery
4) How do you manage this patient?

Mild PE
T. Methyldopa 250mg tds, max
3g/day or
T. Labetolol 100 mg tds, max 300mg
tds
Or, Tab. Nifedipine 10 mg tds stat
dose

Answer
Provisional diagnosis
Pre eclampsia
- Occur at 24w of gestation.
- Primigravida
- Twin pregnancy
- Symptoms of impending pre eclampsia
(frothy urine suggestive of proteinuria
and headache.)

Severe PE
IV hydrallazine start 5mg, double if
no effect until 35mg. change drug if
fails or
IV Labetolol start 10 mg, double if
no effect until max 300mg/day)

Twin to twin transfusion syndrome


-

** MgSo4 slow infusion 4g 10-15


minutes. Maintenance dose IV
ig/hour

Intrauterine blood transfusion from


donor twin to another recipient twin
Donor twin has smaller size and anemic
Recipient twin will be plethoric
Only occur in monozygotic twin with
monochorionic placenta

3) Fetal surveillance
- CTG for fetal well being.
- Biophysical
profile
(Ultrasound
monitoring of fetal movement, fetal tone
and
fetal
breathing,
ultrasound
assessment of liquor volume with or
without assessment of fetal heart rate)

Factors contributes to preterm delivery


-

Lower and upper genital tract infection


Uterine over distension
Cervical incompetence
Maternal
medical
complications,
maternal stress
Fetal, placental or uterine abnormalities

4) Anticipating in preterm delivery by


giving IM Dexamethasone, 12 MG, and
12 hours apart.
5)
38

Others
Bed rest
Reduce physical activity
Reduce high cholesterol and salty diet.

Long Case Examination for Phase III Medical Students


University Science Malaysia

Multiple Pregnancies
Summary of recommendation from Clinical
Management
Guidelines
for
ObstetricianGynecologists Number 56, October 2004

Level B Evidence
Tocolytic agents should be used judiciously in
multiple gestations

Cerclage, hospitalization, bed rest, or home


uterine activity monitoring has not been studied
in high order multiple gestations, and, therefore
should not be ordered prophylactically. There
currently is no evidence that their prophylactic
use improves outcome in these pregnancies

Women with high-order multiple gestations


should be queried about nausea, epigastric pain
and other unusual 3rd-trimester symptoms
because they are at increased risk to develop
HELLP syndrome, in many cases before
symptoms of preeclampsia have appeared.

Because the risks of invasive prenatal diagnosis


procedures such as amniocentesis and chorionic
villus sampling are inversely proportional to the
experience of the operator, only experienced
clinicians should perform these procedures in
high-order multiple gestation.

The higher incidence of gestational diabetes and


hypertension in high-order multiple gestations
warrants screening and monitoring for these
complication.

Women should be counseled about the risks of


high order multiple gestation before beginning
ART

Level C (expert opinion)


The national Institutes of Health recommends
that women in preterm labor with no
contraindication to steroid use be given one
course of steroids regardless of the number of
fetuses

Management of discordant growth restriction of


death of one fetus in a high-order multiple
gestations should be individualized, taking into
consideration the welfare of the other fetus (es)

39

Long Case Examination for Phase III Medical Students


University Science Malaysia
Prop up the patient 45o.
Oxygen 100% 3L/min via nasal prong
Intravenous access.
Take the blood for arterial blood gas
(ABG).
6. 12 lead ECG.
7. Intravenous furosemide, 40 80 mg stat
and maintenance.
8. Intravenous digoxin and intravenous
diamorphine 2.5 5 mg slowly
depending on medical request.
2.
3.
4.
5.

36 years old Malay lady, housewife, G5P4 at


30W+ 5/7 POA with known history of chronic
rheumatic heart disease and on single drug
therapy presented with sudden onset shortness
of breath on the day of admission, cannot lie flat
on night and reduce effort tolerance. There was
no sign and symptoms suggestive of lung
infection. Currently she is not in labour and
fetal movement is good.
Questions
1) What sign and symptoms that you
would like to elicit to suggest that this is
a case of heart failure
2) How do you manage this patient

Further management:
1. Admit the patient to antenatal ward.
2. Carry out all the investigations as
mentioned above.
3. Continue oxygenation.
4. Prop up the patient 45o.
5. Close monitoring of
a) vital signs
b) input output chart
c) Cardiotocography ( CTG )
6. Continue IV furosemide. May change to
tablet form when necessary.
7. Tab slow potassium.
8. Consider IM dexamethasone 12 mg b.d,
6 hours apart to anticipate pre term
delivery.
9. Should be managed together with
medical team.

Answer
Sign and symptoms of heart failure
Cardiac symptoms: exertional dyspnoea,
orthopnea, paroxysmal nocturnal dyspnoea,
dyspnoea at rest, acute pulmonary edema, chest
pain, palpitation.
Non cardiac symptoms: anorexia, nausea, weight
loss, bloating, fatigue, weakness, oliguria,
nocturnal, and cerebral symptoms

Physical examination: Clubbing, prolong


capillary refilling, weak, rapid, and thready
pulse,
tachycardia,
diaphoresis,
pallor,
peripheral cyanosis with pallor and coldness of
the extremities, pulmonary rales, edema,
hepatomegally, pleural effusion, ascites,
cardiomegally, murmurs.

Advice on discharge:
1. Semi bed rest at home and avoid
vigorous activity
2. Regular follow up at combined clinic.
3. If the patient develops any symptoms of
urinary tract infection, upper respiratory
tract infection or chest infection, come
early to the hospital.
4. Advise to deliver in the hospital.
5. Admit when the patient at term OR
admit earlier if the patient has symptoms
of HF.

Management to this patient


Acute management
Similar to managing non pregnant patient where
the aim is to stabilize the patient
1. Secure the ABC airway, breathing and
circulation.
40

Long Case Examination for Phase III Medical Students


University Science Malaysia
29 years old Malay lady, G3P2 at 27/52 W POA
with mitral valve prolapses (not in failure)
3.
Questions
1) What is Eisenmengers syndrome
2) How do you manage this patient
4.
Answer
Heart disease complicates approximately 1% of all
pregnancy with chronic rheumatic heart disease is
the commonest cause in Malaysia apart from
congenital
heart
disease,
cardiomyopathies,
myocarditis and coronary artery diseases.

5.
6.

Eisenmengers syndrome
Pulmonary
hypertension
secondary
to
uncorrected
congenital
heart
disease
characterized with right-to-left shunting and the
associated cyanosis

7.

8.
9.

Management to this patient


Antenatal
1. Should be managed in combined clinic.
2. Advice on
a) Avoid doing vigorous activities
b) Avoid from getting infection i.e.
good oral hygiene, treat UTI or
URTI.
3. Medication T.frusemide, hematinic,
folic acid and anticoagulant prophylaxis
i.e. LMWH
4. Come to the hospitals if develops any
symptoms of heart failure, UTI, URTI
or chest infection
5. Advise to deliver in the hospital.
6. Admit when the patient at term OR
admit earlier if the patient has symptoms
of HF.

withstand stress of normal labour. The


same thing goes for Induction of labour.
Patient should be placed on high risk
category with present of senior
obstetrician and consultation from
cardiologist.
Close monitoring of vital signs, CTG
and oxygen saturation. Each patient
must be attended by one staff nurse or
mid wife
Patient must be in prop up position with
oxygen is freely available.
Pain management in form of epidural
anesthesia (if the patient has no
contraindication) or opiates.
Prophylactic antibiotic to give adequate
protection: Ampicillin or Gentamicin or
Amoxicillin
Prolonged second stage labour must be
assisted
Syntocinon is used in third stage of
labour instead of ergometrine or
syntometrine.

Management of postpartum:
1. Adequate rest for maternal.
2. Encourage breast feeding unless she
cannot cope with it.
3. Continue oral antibiotic for 5 days.
4. Adequate anti coagulant prophylaxis i.e
warfarin to prevent deep vein
thrombosis and thromboembolism.
5. Discussion about family planning.
Based on
a. Severity of the heart disease
b. Completed family
Methods:
a. Hormonal contraception - COC will
increase risk of TE.
b. IUCD should be discouraged due to
risk of infection.
c. Sterilization

Management of intrapartum:
1. Aim for SVD
2. C-sec if any obstetric problem or in
view of cardiologist that patient cannot
41

Long Case Examination for Phase III Medical Students


University Science Malaysia
19 Years old Malay lady, G2P0+1(abortion) at
39 W POA with history of chronic rheumatic
heart disease with mitral valve replacement,
infective endocarditis and completed treatment
and history of overwafarinization at 15W POA
currently admitted for elective heparin infusion

microcephaly, optic atrophy, and blindness,


seizures, Dandy-Walker syndrome, and focal
cerebellar atrophy),absent or non-functioning
kidneys, anal dysplasia, deafness, hemorrhagic
complications, premature births, spontaneous
abortions, stillbirths, and death

Questions
1) Should warfarin be used in pregnancy
2) How to access functional cardiac status
during pregnancy
3) Why do you think this patient admitted
for heparin infusion?

How to access functional cardiac status during


pregnancy

Answer
Should warfarin be used in pregnancy?
Warfarin (Coumadin) is an oral anticoagulant
that inhibits synthesis of vitamin Kdependent
clotting factors, including factors II, VII, IX, and
X, and the anticoagulant proteins C and S
[Shirin Abadi et al]
Literature suggests a strong association between
maternal warfarin use and fetal adverse effect
[Shirin Abadi et al]
If possible, warfarin therapy should be avoided
during pregnancy. If warfarin therapy is
essential, it should be avoided at least during the
first trimester (because of teratogenicity) and
from about 2 to 4 weeks before delivery to
reduce risk of hemorrhagic complications
[Shirin Abadi et al]

Clinical Practice Guideline on "Management of


Heart failure", KKM
Why do you think this patient admitted for
heparin infusion?
Pregnancy itself predispose patient to risk of
thromboembolism. Furthermore, this patient has
been on prosthetic mitral valve which further
increases the risk of thromboembolism.

Using warfarin between 6 and 12 weeks


gestation is associated with fetal warfarin
syndrome, which is most commonly manifested
by nasal hypoplasia, stippled epiphyses, limb
deformities, and respiratory distress

Unfractionated heparin or low molecular weight


heparin could be substituted when appropriate
because these agents do not cross the placenta
and are considered the anticoagulant drugs of
choice during pregnancy. [Shirin Abadi et al]

Other complication includes central nervous


system abnormalities (mental retardation,
42

Long Case Examination for Phase III Medical Students


University Science Malaysia
due to any structural or functional cardiac
conditions [Hunt SA et al]

38 Years old Malay lady, housewife,


G9P6+2(abortion) at 12/52 POA and known
case of Mitral Valve Prolapse with mild Mitral
regurgitation for more than 10 years

According to European Society of Cardiology,


HF is a syndrome whereby the patient should
have the following features; typically shortness
of breath at rest or exertion, and/fatigue sign,
signs of fluid retention and objective evidence of
an abnormality of the structure or function of the
heart at rest.

Questions
1) Type of murmur in mitral regurgitation
and stenosis and Pathophysiology
2) What is heart failure?
3) Contraindication for pregnancy in heart
disease

There have been a confusing in defining the type


of heart failure especially involving the acute or
chronic state. Furthermore, many clinicians use
it interchangeably to refer to severity of the
disease. Therefore, the ESC guidelines have
come across with new definition which
distinguishes between new onset HF, transient
HF and chronic HF.

Types of murmur and Pathophysiology


Mitral regurgitation
- Pan systolic murmur
- Occur when ventricles leak to a lower
chamber or vessel because there is
pressure gradient from the moment
ventricle begin to contract.
- Blood then flow and murmur begin at
beginning of first heart and continue
until the pressure equalize
- Other causes of pan systolic murmur
include tricuspid regurgitation, VSD,
and Aortopulmonary shunts.

New onset HF is self-explanatory and refers to


first presentation.
Transient HF refer to symptomatic HF over a
limited time period, although long-term
treatment may be indicated, for examples;
patient with mild Myocarditis with nearly
complete recovery, patient with MI who needs
diuretic in CCU but not require it in long term
treatment or transient HF caused by ischemia
that resolve with revascularization

Mitral stenosis
- Mid diastolic murmur.
- Begin later in diastolic and may be short
or extend right up to first heart sound.
- Due to impairs flow during ventricular
filling either because of stenosis, or
obstruction by tumor mass (atrial
myxoma)
- Can also due to Austin Flint murmur of
aortic regurgitation and Carey Coombs
murmur of acute rheumatic fever,.

Meanwhile chronic heart failure is a persistent


heart failure with stable, worsening or
decompensated state.

What is Heart failure?

1)
2)
3)

Heart failure is a syndrome manifesting as the


inability of the heart to fill with or eject blood

4)
5)

43

Contraindication for pregnancy


Pulmonary hypertension
Eisenmengers Syndrome
Aortic dilatation > 4cm and this should
be suspected in Marfan syndrome
Mother on warfarin treatment.
Severe cyanotic heart disease with low
oxygen saturation and high hematocrit

Long Case Examination for Phase III Medical Students


University Science Malaysia
6) Excludes the differential diagnosis of
lethargy, shortness of breath and lightheadedness
a) Cardiovascular problem
b) Respiratory tract infection
c) Multiple pregnancy
d) Diabetes mellitus
7) For iron deficiency anemia
a) Iron supplement like ferrous
sulphate and ferrous fumarate
(increase approximately 1 Hb in 1
week)
# may consider double hematinic
(doubling the dose)
b) Parenteral iron
c) Packed cell transfusion.
8) Maternal
transfusion
should
be
considered for fetal indications in cases
of severe anemia.
9) Course of treatment should be based on
clinical judgment and individualized;
period of gestation, severity of anemia,
type of anemia.

18 years old Malay lady, G1P0 at 32/52 POA


was admitted because of severe lethargy,
shortness of breath and light-headedness.
Questions
1) Define anemia
2) Common cause of anemia in pregnancy
3) Management to this lady
4) What is haematinic
5) 1 pack cell blood can increase how
much hemoglobin level
6) Complication of anemia

Answer
Anemia
Hemoglobin concentration <11.0 g/dL [WHO]
a) Mild (Hb 8-10 g/dL)
b) Moderate (Hb 5-8 g/dL)
c) Severe (Hb less than 5 g/dL
Common cause of anemia in pregnancy
1) Microcytic anemia (Iron deficiency
anemia. Needs to exclude Thalassemia
as both will give low MCV of <85 fL)
2) Macrocytic anemia (folate deficiency)
3) Trauma
4) Hemolytic anemia
a) Sickle cell syndrome
b) Sickle cell disease
c) Sickle cell traits
d) Sickle cell hemoglobin C disease.

Haematinic
a) Iron
b) Folate
c) Vitamin C (increase absorption of iron)

One packed cell


Is 450 ml of blood. Ideally it will
increase 1 Hb level

Complication of anemia
1) To mother
a) Aggravate heart failure
b) Risk of post partum hemorrhage
c) Increase risk of infection

Management for this lady


1) Admit to wards for observation
2) Monitoring of vital sign.
3) Full blood count (pay attention on
hemoglobin level for grading and MCV
for type of anemia), serum ferritin level,
total iron binding capacity.
4) Full blood picture if iron deficiency
anemia is unlikely
5) Establish the causes of anemia

2) To fetus
a) Fetal hypoxia
b) IUGR
c) Spontaneous abortion.

44

Long Case Examination for Phase III Medical Students


University Science Malaysia

Clinical Management Guidelines for ObstetricianGynecologists


Number 95, July 2008

Anemia: Hgb (g/dL) and Hct (percentage)


levels below 11 g/dL and 33%, respectively,
in the first trimester; 10.5 g/dL and 32%,
respectively, in the second trimester; and 11
g/dL and 33%, respectively, in the third
trimester
Summary of
Conclusions

Recommendations

The following recommendations are based


primarily on consensus and expert opinion
(Level C):

Iron supplementation decreases the


prevalence of maternal anemia at
delivery.

The following
recommendation
and
conclusions are based on limited or
inconsistent scientific data (Level B):
-

All pregnant women should be


screened for anemia, and those with
iron deficiency anemia should be
treated with supplemental iron, in
addition to prenatal vitamins.

Patients with anemia other than iron


deficiency anemia should be further
evaluated.

Failure to respond to iron therapy


should prompt further investigation
and may suggest an incorrect
diagnosis,
coexisting
disease,
malabsorption (sometimes caused by
the use of enteric-coated tablets or
concomitant use of antacids),
noncompliance, or blood loss.

and

The following conclusion is based on good


and Consistent scientific evidence (Level A):
-

Iron deficiency anemia during


pregnancy has been associated with
an increased risk of low birth weight,
preterm delivery, and perinatal
mortality. Severe anemia with
maternal Hgb levels less than 6 g/dL
has been associated with abnormal
fetal oxygenation resulting in non
reassuring fetal heart rate patterns,
reduced amniotic fluid volume, fetal
cerebral vasodilatation, and fetal
death. Thus, maternal transfusion
should be considered for fetal
indications.
45

Long Case Examination for Phase III Medical Students


University Science Malaysia
28 Years old Malay lady, housewife, G2P1 at
38W + 5/7 POA was admitted in view of

Management of pregnant lady with fibroids


1) Management
of
fibroids
during
pregnancy is just a monitoring of its
growth. Drugs rarely being prescribed.
2) Fibroids can cause miscarriage.
Therefore, advise patient not to
undergone vigorous activity.
3) Corticosteroid injection if pre term
labour is anticipated.
4) Monitoring of the fetus for fetal well
being, lie and presentation.
5) Observation of fetal lie and presentation
is crucial before allowing patient to go
for SVD.
6) Furthermore, obstructed labour should
be excluded.
7) If patient is scheduled for caesarean
section, do not remove the fibroid
during the surgery as it will cause heavy
bleedings.
8) Fibroids will shrink after pregnancy.
Management post pregnancy includes
a) Ablation of the fibroids
b) Surgery to remove the fibroids
c) Hysterectomy
d) Medications to shrink the fibroids

1. One Previous scar a year ago due to


fetal distress, uncomplicated CS
2. Uterine fibroid diagnosed during first
pregnancy
Questions
1) Why fibroid increase in size during
pregnancy
2) What fibroid changes can occur during
pregnancy
3) Expected findings during PE
4) How do you manage the pregnancy
patient with fibroids?
Answer
Why fibroid increase in size during pregnancy
Fibroid is an estrogen-dependant for its growth.
During pregnancy, the level of estrogen rise
steadily until term.
Fibroid changes during pregnancy
Red degeneration can occurs between 12th and
22nd week of pregnancy where the blood supply
to the fibroid is cut off. As a result, the fibroid
turn red and die. Red degeneration can cause
intense abdominal pains and uterine contraction
which could lead to early labour or miscarriage
Expected findings during PE
1) More than 2 fetal pole palpated
2) Some fibroids may cause unstable lie or
breech presentation.

46

Long Case Examination for Phase III Medical Students


University Science Malaysia
e) Leading causes of death for girls aged
15 to 19 in developing countries
f) Predispose to un safe abortion
g) High risk for HIV infection due to
unprotected sex

16 years old Malay lady, single parent with


G1P0 at 37 w + 6/7 POA with history of
1) Bronchial asthma on MDI salbutamol
and MDI inflammide
2) Anemia on double hematinic. History of
blood transfusion at 26/52
3) Ante partum hemorrhage secondary to
PP type I at 34/52. Completed
dexamethasone.
4) Currently
admitted
because
of
contraction pain.
5) Pre marital sex

Dexamethasone
Pre term labor associated with complication of
respiratory distress syndrome, intraventricular
hemorrhage and necrotizing enterocolitis.
Multiple randomized controlled trials has
demonstrated
that
the
admission
of
corticosteroid to the mother resulting significant
reduction in these complication.
Both betamethasone and dexamethasone can
cross the placenta. In USM, dexamethasone is
used instead of betamethasone. Betamethasone
may be a better choice because it can reduce the
risk of cystic periventricular leukomalacia.

Questions
1) What is the effect of Salbutamol
(ventolin) on uterine contraction
2) Complication of teenage pregnancy
3) Principles of Dexamethasone and
dosage.

It can be given in doses of 6 mg every 12 hours


for four doses, and 12 mg every 12 hours for 2
doses during period of gestation of 24 W to 34
through IM route.

Answer

In term of lung maturity, no benefit has been


demonstrated in infants beyond 34 weeks
gestation. However, it is still being given in
many centers because of no harm to the baby
and mother. Furthermore, it still can improve the
complication of intraventricular hemorrhage and
necrotizing enterocolitis.

Salbutamol action on uterus


Salbutamol stimulates beta 2 receptor in the
uterus and causing muscles in the wall of uterus
to relax.
Complication of teenage pregnancy
Teenage pregnancy is defined as a teenage girl,
usually within the ages of 13-19, becoming
pregnant. [UNICEF]

Beware when used in mother with poorly


controlled
diabetes
in
pregnancy,
chorioamnionitis
and
immunosuppressed
mothers

a) Highest global incidence for premature


birth and low birth weight
b) High risk for anemia in pregnancy
c) Difficulties
in
labour
due
to
underdeveloped pelvis
d) High risk for obstructed labour, causing
obstetric fistula if c-sec is not readily
accessible

Corticosteroid shows maximum effect after 24


hour and lasted 7 days. Repeated use of
corticosteroid should be avoided as it may
impose complication to mother and fetus.

47

Long Case Examination for Phase III Medical Students


University Science Malaysia
2) Counseling on mode of delivery and
reason why patient can not be allowed
for SVD.
3) Plan for Caesarean section. Can be
performed at 38 week to 39 week of
gestation.
4) Counseling for bi tubal ligation because
patient is in high risk to develop uterine
rupture, hemorrhage and uterine atony
after this c-sec delivery due to
a) Grand multi para
b) 1 c-sec scar.

33 Years old Malay lady, housewife, G7P6 at 38


W POA was admitted for further management in
view of
1) Grand multi para
2) Maternal obesity but MOGTT test is
normal
3) Placenta previa type III anterior but no
history of APH.
4) Biggest baby is 3.9 kg.
5) Clinically, suspected macrosomic baby.
On ultrasound, estimated fetal weight is
4.0 to 4.2 kg.
6) Not in labour yet.

Intrapartum management
1) Preparation for c-sec (refer c-sec
preparation)
2) Blood GSH because anticipating in
blood loss because we will cut through
the placenta.
3) Presence of senior obstetrician in case of
complication to mother during operation
and pediatrician for management of
baby.

Question
1) What is the mode of delivery and justify
your answer
2) How do you manage this patient
Answer
Mode of delivery
After thorough view on this patient presentation,
caesarian section is the most appropriate mode
of delivery because of

Post partum
1) Baby should be check for capillary
blood sugar and early feeding is
encouraged
2) Baby should be managed by pediatrician
and kept in NICU for observation.
3) Mother should be offered with other
type of contraception if she refused bi
tubal ligation.
4) Daily inspection on c-sec scars to look
for any infection or ruptured scar.
5) Referral to the mother to dietitian and
internal medicine team for further
management on obesity.
6) Mother should be offered MOGTT
screening on the next pregnancy.

1) Placenta previa type III in which


descending of fetal head into pelvic
brim is impossible due to obstruction to
cervical os.
2) Maternal obesity and macrosomic baby
will
predispose
to
complication
especially shoulder dystocia.
3) Induction of labour also impossible
because of grand multi para with
macrosomic baby.
Management to this patient
Ante natal management
1) Daily CTG for fetal surveillance.
Ultrasound should also be done.

48

Long Case Examination for Phase III Medical Students


University Science Malaysia
than in the multiparous group (P < .05). Similar
frequency of maternal diabetes, infection,
uterine wall scar rupture, variations in fetal heart
rate, fetal death, and neonatal mortality was
found in the 3 groups. [Agota Babinszki et al]

40 Years old Malay lady, housewife, G15P12+2


(abortion) at 36W + 3/7 from low
socioeconomic status presented with this
problem list
1) Elderly pregnancy
2) Great grand Multipara (para >10)
3) Unstable lie

Principles of management in this patient


1) Daily observation for fetal lie
2) Exclude factors contributing to unstable
lie such as fetal abnormality, placenta
previa, uterine abnormality
3) Pelvic abnormality
4) Polyhydramnios
5) Discuss on mode of delivery which is
expectant vs. active management (C-sec,
ECV or stabilizing induction)
6) Advise on contraception. Tubal ligation
should be offered in view of
a) Advanced maternal age
b) Great grand multi parity
c) Two history of abortion.
d) Low socio economic status.
 As a Muslim, permanent method of
contraception should only be performed
if specialist agrees that next pregnancy
will be harmful to the patients life.

Questions
1) Complication associated with great
grand Multipara
2) Principles of management in this patient
3) What is the contraindication for ECV?
Answer
Notes: The risk of cord prolapses leading to fetal
hypoxia and fetal death is very high once the
membrane ruptures. Therefore, it is highly
recommended that patient with unstable lie
should be admitted to antenatal wards at 37
weeks onward for observation [The Practical
Labour suite Management]
Complication associated with great grand
Multipara
The incidence of malpresentation at the time of
delivery, maternal obesity, anemia, preterm
delivery, and meconium-stained amniotic fluid
increased with higher parity, whereas the rate of
excessive weight gain and cesarean delivery
decreased. Compared with grand multiparas,
great-grand multiparas had significantly elevated
risks for abnormal amounts of amniotic fluid,
abruptio placentae, neonatal tachypnea, and
malformations but lower rates of placenta
previa (P < .05). The incidence of postpartum
hemorrhage, preeclampsia, placenta previa,
macrosomia, postdate pregnancy, and low Apgar
scores was significantly higher in grand
multiparas than in multiparas, whereas the
proportion of induction, forceps delivery, and
total labor complications was significantly lower

Contra indication for ECV


Absolute contraindication
1) Multiple pregnancy
2) APH
3) PP
4) PROM or PPROM
5) Significant fetal abnormality
6) Any indication for caesarean section.
Relative
1) Previous c-sec
2) IUGR
3) Severe proteinuric hypertension
4) Rhesus iso-immunization
5) Evidence of macrosomia
6) Any suspected fetal compromise.

49

Long Case Examination for Phase III Medical Students


University Science Malaysia
a) Severe cognitive, neurological
developmental activity

26 Years old Malay lady, G1P0 at 38 W POG


currently presented with this problem
1. Extended breech
2. Persistent proteinuria
3. Hypothyroidism on Tab Thyroxine 50
microgram OD and undergone total
thyroidectomy in 2002

Management to this lady


1) The management in view of breech is
similar to normal pregnancy
2) Excludes the causes of proteinuria. Take
notes that primid with proteinuria and
hypothyroidism
should
increase
suspiciousness to pre eclampsia even
though it is already near term.
3) For the management of hypothyroidism
a) WHO recommends intake of 200
micrograms/day of iodine during
pregnancy to maintain adequate
thyroid hormone production
b) Dosage of maintenance thyroxine
could be increased during pregnancy
up to 50-200 microgram daily dose.
(Doubling the dose by 25% to 50%)
c) Thyroid function tests every 6-8
weeks during pregnancy to ensure
normal thyroid function throughout
pregnancy.
4) Screening
for
congenital
hypothyroidism to the baby using the
umbilical cord blood during delivery.

Questions
1) Hormones affecting level of thyroid
hormone during pregnancy.
2) What is the effect of hypothyroidism in
pregnancy
3) Management to this lady

Answer
Hormones interacting with thyroid hormones
1) High level of human chorionic
gonadotropin will decrease the level of
TSH during first trimester
2) Estrogen will increases the amount of
thyroid hormone binding proteins in the
serum hence increases the total thyroid
hormone levels in the blood. However,
free hormone remains normal
Effect of hypothyroidism on pregnancy
1) Mother
a) No
symptoms
in
hypothyroidism
b) Maternal anemia
c) Maternal myopathy
d) Congestive heart failure
e) Pre eclampsia
f) Placental abnormalities
g) Low birth weight infants
h) post partum hemorrhage

and

mild

2) Baby
50

In Pursuit to Excel MCQ Exam for Professional III Examination (MCQ)


Ashermans
syndrome
adhesion) is associated with:
A)
B)
C)
D)
E)

Fibroids, due to their mere presence, cause the


entire uterus to enlarge, thereby stretching the
blood vessels that supply the various parts of the
uterus. When the fibroids are removed, the
remaining uterus collapses down to a smaller
volume and some of the blood vessels that
supply the endometrium may become blocked.
Because of the lack of oxygen and nutrients, that
area of tissue may die and a scar may form
leading to Ashermans syndrome.

(intrauterine

Amenorrhea
Placenta previa
Subfertility
Salphingitis
Menorrhagia

History: Named after Dr Asherman, an Israeli


gynaecologist, who first described the condition
in the mid 20th century when he noted that some
women who had surgical treatments at the
time of pregnancy stopped having periods
after this treatment

Other cause: radiation cause ischemia to


myometrium tissue
Signs and symptoms
1) Oligomenorrhea, amenorrhea
2) Pain (increase work by uterine muscle to get
rid blood through scar tissue)
3) Infertility
4) Haematometra (large bruise inside uterus that
diagnosed by pelvic US)

Def: A reduction or absence in menstruation


that may be due to scar tissue formation inside
the uterus and can occur as a result of
pregnancy and delivery, infection or
gynaecological surgical procedure

Complication
1) Placenta previa, accrete
2) Infertility
3) IUGR
4) Ectopic pregnancy

**differ from endometrial ablation: induce


scar tissue in the uterus to prevent heavy periods
Classification according to 17th Congress of the
Federation of Frenchspeaking Societies of
Gynaecology and Obstetrics [1957]
(1) Traumatic synechiae connected with surgical
or obstetrical evacuation of the uterus.
(2) Spontaneous synechiae of tuberculous origin.
(3) Synechiae occurring after myomectomy.
(4) Synechiae secondary to the attack of
chemical or physical agents and, likewise, those
resulting fromatrophic changes.

Investigation:
1) Hysteroscopy
2) Hysterosalpingogram (HSG)
Management
1) Admission to wards
2) Investigation to confirm the diagnosis
3) Surgical excision of scar tissue by
hysteroscope under General anesthesia.

Pathophysiology: Severely damaged decidua


basalis are replaced with granulation tissue and
opposing uterine wall adhere to form scar tissue.
It is later infiltrated by myometrial cells and
covered by endometrium.

Answer: T, T, T, F, F

In Pursuit to Excel MCQ Exam for Professional III Examination (MCQ)


a. Non pharmacological (mild)
- limiting strenuous exercise
- adequate rest
- maintaining a low salt diet
- treating anemia and infections early
- frequent antenatal examinations

Concerning heart disease in pregnancy


A. Mitral stenosis carries a better
prognosis than atrial defect
B. Pregnancy should be induced at 38
weeks in cases with grade IV
dyspnoea
C. Rheumatic heart disease is more
common than congenital heart disease
D. Ergometrine should be avoided in
most cases
E. Mitral volvotomy is contraindicated.

Pharmacological
- Sublingual GTN
- Digoxin
- Diuretic(used with care as may impair uterine
blood flow. No teratogenic effect)
- Beta blocker (used with care;intrauterine
growth retardation, apnea at birth,fatal
bradycardia,
hypoglycaemia
and
hyperbilirubinemia)
- ACEI and ARB are contraindicated in
pregnancy.
-warfarin is teratogenic in early trimester.
Heparin can be used LMW subcutaneous.

Introduction: About 0.5 4% of pregnant


women have cardiac disease. Common causes of
HF in pregnancy are hypertension, eclampsia,
undetected valvular heart disease especially
mitral stenosis, congenital heart disease, and
occasionally peripartum cardiomyopathy.
Peripartum cardiomyopathy occurs in 1:3,000
life births in Malaysia [Management of HF
Malaysia]

Labour is spontaneous except in fetal


compromise (consider pre mature delivery)

NYHA classification did not show any marked


differences in outcome.

Epidural anaesthesia
recommended.

Eismengers
syndrome
and
pulmonary
hypertension carries 40-50% mortality rate (can
be caused by mitral stenosis). TOF 5% if no
pulmonary HPT.

during

labour

is

Ergometrine causes 1. Vasoconstriction, HPT


and heart failure. So must be avoided. Used
syntocinon only.

Normal haemodynamic changes that occur in


pregnancy are:
1) Cardiac output increases by 3050% during
normal pregnancy.
2) Cardiac output increases to 80% above
baseline during labour and delivery.

Antibiotic prophylaxis in structural heart


abnormality during labour (IV ampiillin 1.0g 6h
X 3doses, IV gentamycin 80 mg 8 hourly X 3)
- Surgical volvotomy ideally be performed
before pregnancy although it is safe to do it
during pregnancy
- Marfan syndrome is an autosomal dominant
connective tissue abnormality that may lead to
mitral valve prolapsed and aortic regurgitation,
aortic root dilatation and aortic rupture (50% in
pregnancy)

**Haemodynamic changes return to baseline 2


4 weeks after vaginal delivery and up to 6
weeks after caesarian delivery.
Management
1) Manage by multidisciplinary team consist of
physician, obstetrician and pediatrician

Answer: F, F, T, T, F
2

In Pursuit to Excel MCQ Exam for Professional III Examination (MCQ)


Pyelonephritis in pregnancy
A. Occur in 0.1% of pregnant woman
B. If unilateral, is most often right sided
C. Caused predominantly by staph.
Aureus
D. Common in diabetic patient

Investigation
1) Full blood count
2) Urinalysis:Positive results for nitrites,
leukocyte esterase, WBCs, RBCs, and protein
suggest UTI.
3) Urine culture: A colony count of 100,000
colony-forming units (CFUs) per milliliter has
historically been used to define a positive culture
result
4) Renal ultrasonography
5) Evaluation of fetal status
6) Renal function test

Def: Pyelonephritis is the most common urinary


tract complication in pregnant women, occurring
in approximately 2% of all pregnancies.
Symptoms of pyelonephritis include the
following:
1. Fever (Often, the temperature is very high.)
2. Chills
3. Nausea and vomiting
4. Costovertebral angle (CVA) or flank pain

Treatment
-Ampicillin 2 g IV q6h for treatment of
pyelonephritis; use in conjunction with an
aminoglycoside for treatment of pyelonephritis
-Paracetamol
-Amoxicillin 7-Day regimen: 250 mg PO q8h or
3-Day regimen: 500 mg PO qid
-Amoxicillin/clavulanate
potassium
(Augmentin), 500 mg PO tid for 7-10 d
-Ceftriaxone (Rocephin)

Flank tenderness is right-sided in more than half of


patients, bilateral in one fourth of patients, and leftsided in one fourth of patients. Pain may also be
found suprapubically with palpation.

Other symptoms may include nausea, vomiting,


frequency, urgency, and dysuria.

Complication
- Bacteremia
- Respiratory insufficiency due to bacterial
endotoxin damage to the alveoli, causing
pulmonary edema; therefore, fluid overload
- Renal dysfunction
- PPROM
- Pre term birth

Women
with
additional
risk
factors
(immunosuppression, diabetes, sickle cell
anemia, neurogenic bladder, recurrent or
persistent UTIs prior to pregnancy) are at an
increased risk of a complicated UTI
Common organism
1. Escherichia coli (most common, in as
many as 70% of cases)
2. Group B Streptococcus (10%)
3. Klebsiella or Enterobacter species (3%)
4. Proteus species (2%)

Answer; F, T, F, T

Physiological changes include urinary retention


caused by the weight of the enlarging uterus and
urinary stasis due to ureteral smooth muscle
relaxation

In Pursuit to Excel MCQ Exam for Professional III Examination (MCQ)


Answer: T, T, T, F, F

STD include
A. Trichomonas vaginitis
B. Condyloma accuminata
C. Chlamydial infection
D. Type 1 herpes hominis
E. Toxoplasmosis

Bacterial STDs include syphilis, gonorrhea,


chancroid,
lymphogranuloma
venereum,
granuloma
inguinale,
and
chlamydial,
mycoplasmal, and Ureaplasma infections.
Viral STDs include genital and anorectal warts
(Condylomata acuminata :HPV types 6 and 11),
genital herpes, molluscum contagiosum, and
HIV infection
Parasitic infections that can be sexually
transmitted include trichomoniasis (caused by
protozoa), scabies (caused by mites), and
pediculosis pubis (caused by lice).
Type 1 herpes hominis causes classic cold
sores or fever blisters, commonly known as
herpes simplex, herpes genitals, herpes labialis
Toxoplasmosis is caused by infection with
Toxoplasma gondii, an obligate intracellular
parasite. The infection produces a wide range of
clinical syndromes in humans, land and sea
mammals, and various bird species.Individuals
at risk for toxoplasmosis include fetuses,
newborns, and immunologically impaired
patients. Congenital toxoplasmosis is usually a
subclinical infection. Among immunodeficient
individuals, toxoplasmosis most often occurs in
those with defects of T-cellmediated immunity,
such as those with hematologic malignancies,
bone marrow and solid organ transplants, or
AIDS.T gondii oocysts are ingested in material
contaminated by feces from infected cats.
Oocysts may also be transported to food by flies
and cockroaches
4

In Pursuit to Excel MCQ Exam for Professional III Examination (MCQ)


Consequences of Uterine rupture in fetal
1) Fetal hypoxia or anoxia
2) Fetal acidosis
3) Admission to a NICU
4) Fetal or neonatal death

Uterine rupture may be associated with


A. Previous c-section
B. Myomectomy
C. Oxytoxin infusion
D. Prostaglandin administration
E. Breech extraction

Consequences of Uterine rupture in mother


1) Maternal bladder injury
2) Severe maternal blood loss or anemia
3) Hypovolemic shock
4) Need for hysterectomy
5) Maternal death

Uterine rupture in pregnancy is a rare and often


catastrophic complication with a high incidence
of fetal and maternal morbidity. Several factors
are known to increase the risk of uterine rupture,
but, even in high-risk subgroups, the overall
incidence of uterine rupture is low. From 19762005, 19 peer-reviewed publications that
described the incidence of uterine rupture
reported 1654 cases of uterine rupture among
2,504,456 pregnant women, yielding an overall
rupture rate of 1 in 1514 pregnancies (0.07%).

Risk Factor
1) Previous cesarean delivery
2) Previous myomectomy
3) Congenital uterine anomaly
4) Pregnancy considerations
Grand multiparity, Maternal age,
Placentation (accreta, percreta, increta,
previa, abruption)
Cornual (or angular) pregnancy
Overdistension
(multiple
gestation,
polyhydramnios)
Dystocia (fetal macrosomia, contracted
pelvis)
Trophoblastic invasion of the myometrium
5) Labor status
Induced labor
+ With oxytocin
+ With prostaglandins
Augmentation of labor with oxytocin
Duration of labor, Obstructed labor
6) Obstetric management considerations
Instrumentation (forceps use)
Intrauterine
manipulation
(external
cephalic version, internal podalic version, breech
extraction, shoulder dystocia, manual extraction
of placenta)
Fundal pressure
7) Uterine trauma
Direct uterine trauma and Violence

Uterine rupture is defined as a full-thickness


separation of the uterine wall and the overlying
serosa.
Differs from uterine scar dehiscence: Separation of a
preexisting scar that does not disrupt the overlying
visceral peritoneum (uterine serosa) and that does
not significantly bleed from its edges. In addition, the
fetus, placenta, and umbilical cord must be contained
within the uterine cavity, without a need for cesarean
delivery because of fetal distress.

Uterine rupture results in:


bleeding;
rupture of the amniotic sac (bag of
waters);
partial or full delivery of the fetus into
the abdominal cavity; and
loss of oxygen delivery to the fetus.
Classic symptoms of rupture include:
pain above and beyond normal labor
pain;
discontinuation of uterine contractions;
signs of fetal heart rate abnormalities;
hemorrhage; and
Hypovolumic shock

Answer: All True


5

In Pursuit to Excel MCQ Exam for Professional III Examination (MCQ)


Labour may be obstructed by
A) Ovarian tumor
B) Cystocoele
C) Ectopic kidney
D) Distended bladder
E) Vaginal septum
Obstructed labour means that, in spite of strong
contractions of the uterus, the fetus cannot
descend through the pelvis because there is an
insurmountable barrier preventing its descent.
Obstruction usually occurs at the pelvic brim,
but occasionally it may occur in the cavity or at
the outlet of the pelvis. [WHO]
Causes of obstructed labour:
1) cephalopelvic
disproportion
(small pelvis or large fetus)
2) abnormal presentations, e.g.
- brow
- shoulder
- face with chin posterior
- aftercoming head in breech
presentation
3) Fetal abnormalities, e.g.
- hydrocephalus*
- locked twins*
4) abnormalities of the reproductive
tract, e.g.
- pelvic tumour*
- stenosis of cervix or vagina**
- tight perineum.**
* Rarer causes.
** This may be associated with scarring caused
by female genital mutilation, or previous
gishiri cut.

Answer: All true

In Pursuit to Excel MCQ Exam for Professional III Examination (MCQ)


However, without pathologic examination of the
uterus, this determination is not possible.
Uterine leiomyosarcomas are found in
approximately 0.1% of women with leiomyomas
and are reported to be more frequently
associated with large or rapidly growing
fibroids.

During pregnancy and puerperium, fibroid


A) increase in size
B) Undergo red degeneration
C) Become infected
D) May undergo sarcomatous changes
E) Cause post partum hemorrhage
Uterine leiomyomas, commonly known as
fibroids, are well-circumscribed, non-cancerous
tumors arising from the myometrium (smooth
muscle layer) of the uterus. In addition to
smooth muscle, leiomyomas are also composed
of extracellular matrix (i.e., collagen,
proteoglycan, fibronectin). Other names for
these tumors include fibromyomas, fibromas,
myofibromas, and myomas.

The two most common symptoms of fibroids


(also called leiomyomas) are abnormal uterine
bleeding and pelvic pressure.
Leiomyomas are also associated with a range of
reproductive dysfunction including recurrent
miscarriage, infertility, premature labor, fetal
malpresentations, and complications of labor.

Leiomyomas are usually detected in women in


their 30's and 40's and will shrink after
menopause in the absence of post-menopausal
estrogen replacement therapy. (Dependent on
estrogen for growth)

Diagnosis
1. bimanual pelvic examination
2. ultrasonography,
MRI
(magnetic
resonance imagery), and CT
3. Hysterosalpingography,
sonohysterography, and hysteroscopy

Two to five times more prevalent in black


women than white women

Red degeneration: obsolete term for necrosis,


with staining by hemoglobin, which may occur
in uterine myomas, especially during pregnancy;
marked by softening and a red color resembling
partly cooked meat. [stedman]

Leiomyomas are classified by their location in


the uterus. Subserosal leiomyomas are located
just under the uterine serosa and may be
pedunculated (attached to the corpus by a
narrow stalk) or sessile
(broad-based).
Intramural
leiomyomas
are
found
predominantly within the thick myometrium but
may distort the uterine cavity or cause an
irregular external uterine contour. Submucous
leiomyomas are located just under the uterine
mucosa (endometrium) and, like subserosal
leiomyomas, may be either pedunculated or
sessile. Tumors in subserosal and intramural
locations comprise the majority (95%) of all
leiomyomas; submucous leiomyomas make up
the remaining 5%. Others; Cervical,
Intraligamentary (within broad ligament, cause
uteric compression) and Parasitic(attached
outside the uterus, i.e. the bladder)

Medical treatment
1) NSAIDS for dysmenorrhea
2) antifibrotic drug, pirfenidone
3) GnRH agonist (Specifically, uterine
volume has been shown to decrease
approximately 50% after three months
of GnRH agonist therapy.)
- Use to reduce fibroid size few
months before surgery or
- When menopause is within few
months
Surgical treatment
Myomectomy (pt wish to reproduce)
Hysterectomy

Transformation of uterine leiomyomas (benign)


to uterine leiomyosarcomas (malignant smooth
muscle tumors of the uterus) is extremely rare,
and, in fact, many researchers and clinicians
believe this type of transformation never occurs.

Answer: T, T, F, F, T

In Pursuit to Excel MCQ Exam for Professional III Examination (MCQ)


* Multiple gestation (larger surface area of the
placenta)
* Erythroblastosis
* Prior uterine surgery
* Recurrent abortions
* Nonwhite ethnicity
* Low socioeconomic status
* Short interpregnancy interval
* Smoking
* Cocaine use
* Other causes include digital exam, abruption
(pre-eclampsia, chronic hypertension, cocaine
use, etc) and other causes of trauma (eg,
postcoital trauma).

Placenta previa is associated with


A) Painless vagina bleeding
B) Abnormal fetal heart rate
C) Twin pregnancy
D) Android pelvis
E) Diabetes Melitus
Placenta previa involves implantation of the
placenta over the internal cervical os. Variants
include complete implantation over the os
(complete placenta previa), a placental edge
partially covering the os (partial placenta previa)
or the placenta approaching the border of the os
(marginal placenta previa). A low-lying placenta
implants in the caudad one half to one third of
the uterus or within 2-3 cm from the os.

Grade.

A leading cause of third trimester hemorrhage,


placenta previa presents classically as painless
bleeding. Bleeding is thought to occur in
association with the development of the lower
uterine segment in the third trimester. Placental
attachment is disrupted as this area gradually
thins in preparation for the onset of labor. When
this occurs, bleeding occurs at the implantation
site as the uterus is unable to contract adequately
and stop the flow of blood from the open
vessels. Thrombin release from the bleeding
sites promotes uterine contractions and a vicious
cycle
of
bleeding-contractions-placental
separation-bleeding.
Placental migration occurs during the second
and third trimesters, owing to the development
of the lower uterine segment, but it is less likely
if the placenta is posterior or if there has been a
previous caesarean section.

Placenta
Lateral
encroaches on
the lower
uterine segment
but does not
reach the
cervical os.

II

Placenta
reaches the
margin of the
cervical os but
does not cover
it.

III

Placenta
partially cover
the os.

IV

Placenta is
Complete
symmetrically
implanted in the
lower uterine
segment

marginal

Minor

major if
posterior
located

Causes
* Hemorrhaging, if associated with labor,
would be secondary to cervical dilatation and
disruption of the placental implantation from the
cervix and lower uterine segment. The lower
uterine segment is inefficient in contracting and
thus cannot constrict vessels as in the uterine
corpus, resulting in continued bleeding.
* Advancing age (>35)
* Multiparity
* Infertility treatment

Answer: T, F, T, F, F
8

Major

In Pursuit to Excel MCQ Exam for Professional III Examination (MCQ)


corrected to ensure adequate hemostasis in the
case of a cesarean

Complication of abruptio placenta


A) Eclampsia
B) Acute renal failure
C) DIVC
D) Fetal death
E) Post partum hemorrhage

Prematurity: Delivery is required in cases of


severe abruption or when significant fetal or
maternal distress occurs, even in the setting of
profound prematurity. In some cases, immediate
delivery is the only option, even before the
administration of corticosteroid therapy in these
premature infants. All other problems and
complications associated with a premature infant
are also possible.

Abruptio placentae is defined as the premature


separation of the placenta from the uterus.
Patients with abruptio placentae typically
present with bleeding, uterine contractions, and
fetal distress. A significant cause of thirdtrimester bleeding associated with both fetal and
maternal morbidity and mortality, abruptio
placentae must be considered whenever bleeding
is encountered in the second half of pregnancy.

Signs and symptoms


1) Vaginal bleeding
- Vaginal bleeding is present in 80%
of patients diagnosed with placental
abruptions.
- Bleeding may be significant enough
to jeopardize both fetal and maternal
health in a relatively short period.
- Remember that 20% of abruptions
are associated with a concealed
hemorrhage and the absence of
vaginal bleeding does not exclude a
diagnosis of abruptio placentae.
2) Contractions/uterine tenderness
- Contractions and uterine hypertonus
are part of the classic triad observed
with placental abruption.
- Uterine activity is a sensitive marker
of abruption and, in the absence of
vaginal bleeding, should suggest the
possibility
of
an
abruption,
especially after some form of
trauma or in a patient with multiple
risk factors.
3) Decreased fetal movement
- This may be the presenting
complaint.
- Decreased fetal movement may be
due to fetal jeopardy or death.

Hemorrhage into the decidua basalis occurs as


the placenta separates from the uterus. Vaginal
bleeding usually follows, although the presence
of a concealed hemorrhage in which the blood
pools behind the placenta is possible.
If the bleeding continues, fetal and maternal
distress may develop. Fetal and maternal death
may occur if appropriate interventions are not
undertaken. The primary cause of placental
abruption is usually unknown, but multiple risk
factors have been identified.
Complication
Cesarean delivery: Cesarean delivery is often
necessary if the patient is far from her delivery
date or if significant fetal compromise develops.
If significant placental separation is present, the
fetal heart rate tracing typically shows evidence
of fetal decelerations and even persistent fetal
bradycardia. A cesarean delivery may be
complicated
by
infection,
additional
hemorrhage, the need for transfusion of blood
products, injury of the maternal bowel or
bladder, and/or hysterectomy for uncontrollable
hemorrhage. In rare cases, death occurs.
Hemorrhage/coagulopathy:
Disseminated
intravascular coagulation (DIC) may occur as a
sequela of placental abruption. Patients with a
placental abruption are at higher risk of
developing a coagulopathic state than those with
placental previa. The coagulopathy must be

Answer: F(risk factor), T, T, T, T(uterine atony)

In Pursuit to Excel MCQ Exam for Professional III Examination (MCQ)


2. Early Rupture of Membranes (AROM)
3. Internal Fetal Monitoring (fetal scalp
electrode)
4. Tocometry
5. Intrauterine Pressure Catheter
6. Cautious use of Pitocin
E. Risks
1. Preterm birth
2. Intrauterine Growth Retardation

A patient with severe placental abruption will


need
A) CVP line
B) Artificial Rupture Of Membrane
C) Sedation with morphine
D) Beta adrenergic drug
E) Arterial Blood Gas analysis
Sher Severity Grading system
1. Grade 1: (Herald bleed)
1. Less than 100cc of uterine bleeding
2. Uterus non-tender
3. No Fetal Distress
2. Grade 2
1. Uterus tender
2. Fetal Distress
3. Concealed hemorrhage
4. Progresses to Grade 3 without delivery
3. Grade 3
1. Fetal death
2. Maternal shock
3. Extensive concealed hemorrhage
4. Coagulopathy
1. Absent: 3A (66% of patients)
2. Present: 3B (33% of patients)

Management: Emergent
Rapid management is critical as fetal death
occur in up to 30% within 2h. Do not wait for
US as it is clinically diagnosed
1. Brisk bleeding
2. Unstable vital signs
3. Fetal Distress
4. Grade II or III placental abruption
Immediate interventions
1. Oxygen
2. Trendelenburg position
3. Obtain immediate Intravenous Access
1. Two large bore IV (16-18 gauge)
2. Initiate Isotonic crystalloid bolus
1. Normal saline or Ringers
4. Call for immediate Obstetric and
neonatal support
5. Delivery within 20 minutes if Fetal
Distress** Cesarean Section unless imminent
Vaginal Delivery
6. RhoGAM if Maternal blood Rh -ve

Management: Stable patient (Grade I)


A. General
1. Obstetrics Consultation
2. RhoGAM if Maternal blood Rh -ve
B. Criteria
1. Reassuring Fetal Heart Tracing
2. No coagulopathy
3. Normotensive without Preeclampsia
4. Nontender uterus
5. Negative ultrasound with normal AFI
C. Preterm gestation
1. Consider Tocolysis with Magnesium
Sulfate. Contraindicated in all but mild
abruption <34 weeks**Controversial and risky
2. Steroids to promote lung maturity
3. Consider Amniocentesis for lung
maturity studies
4. External Fetal Monitoring
5.
Observe
during
short
term
hospitalization
D. Term gestation or mature lung studies
1. Active management labor towards rapid
fetal delivery

Monitoring
1. Orthostatic Blood Pressure and pulse
2. Monitor Intake and output
*Keep Urine Output over 30cc per hour
3. Monitor Hemoglobin or Hematocrit q12 hours prn
1. Keep Hemoglobin >10 g/dl or
Hematocrit >30%
2. Packed Red Blood Cell transfusion
as needed
4. Monitor coagulation studies
1. Fresh Frozen plasma transfusion as
needed
2. Platelet transfusion as needed
Answer: T, F(once pt stable), T, F, T
10

In Pursuit to Excel MCQ Exam for Professional III Examination (MCQ)


Twin-to-twin transfusion syndrome (TTTS) is
the result of an intrauterine blood transfusion
from one twin (donor) to another twin
(recipient). TTTS only occurs in monozygotic
(identical) twins with a monochorionic placenta.
The donor twin is often smaller with a birth
weight 20% less than the recipient's birth
weight. The donor twin is often anemic and the
recipient twin is often plethoric with hemoglobin
differences greater than 5 g/dL. [EMedicine
article 271752]

The following are associated with placental


insufficiency
A) Diabetes Mellitus
B) Post maturity
C) Twin pregnancy
D) Cigarette smoking
E) Dieting during pregnancy
Definition:
Placental insufficiency is the failure of the
placenta to supply nutrients to the fetus and
remove toxic wastes.
In post maturity of the baby [before term,
growth of placenta is proportional to growth of
fetus. However after term, placenta start to
regress while the baby continue to grow]

Answer: T, T, T, T, T

Causes [from Ten Teachers]


Reduced uteroplacental perfusion: Inadequate
trophoblast
invasion,
anti
phospholipid
syndrome, diabetes mellitus, Sickle cell disease,
multiple gestation, collagen vascular disease.
This will result in small placenta with gross
morphological changes. Usually infarct and
basal hematoma.
Reduce feto placental perfusion: Single
umbilical
artery,
twin-twin
transfusion
syndrome.
Antiphospholipid syndrome (APS) is a disorder
that manifests clinically as recurrent venous or
arterial
thrombosis
and/or
fetal
loss.
Characteristic laboratory abnormalities in APS
include persistently elevated levels of antibodies
directed
against
membrane
anionic
phospholipids
(ie, anticardiolipin [aCL]
antibody, antiphosphatidylserine) or their
associated plasma proteins, predominantly beta2 glycoprotein I (apolipoprotein H); or evidence
of a circulating anticoagulant. [EMedicine
article 333221]

11

In Pursuit to Excel MCQ Exam for Professional III Examination (MCQ)


Hydatidiform mole is associated
increased urinary output of
a)
b)
c)
d)
e)

Failure of the placental-site trophoblastic tumor


to produce large amounts of estrogen, in contrast
to normal pregnancy and Hydatidiform mole,
resulted
in
marked
androgen/estrogen
imbalance, high circulating concentrations of
free testosterone, and Virilization. [Nagelberrg
SB& Rosen SW, 1985] A decrease of E3 in
these abnormal pregnancies would result mainly
in a lower level of tissue P450arom
concentration [Takara Yamamoto et al, 1997]

with

estrogen
human chorionic gonadotrophin
prostaglandin
pregnanediol
human placental lactogen

Gestational trophoblastic disease encompasses


of several disease processes that Originate from
placenta
a) Complete mole (no fetal tissue, 90% 46
XX and 10% 46 XY)
b) Partial mole (69, XXX or 69, XXY with
fetal tissue present)
c) Placental site trophoblastic tumors
d) Choriocarcinomas
e) Invasive moles

HPL Value increased in multiple pregnancies


(twins or more), Placental site trophoblastic
tumor, intact molar pregnancy, Diabetes, Rh
incompatibility and deccreased in Toxemia,
Aborting hydatidiform mole, Choriocarcinoma
and Placental insufficiency [Medline Plus]
PGDH activity in neoplastic tissues was found
to be one tenth or less of that in normal term
human placentae. PGDH may be important in
the accumulation of PGs in neoplastic tissues. If
choriocarcinoma cells were able to synthesise
prostaglandin E2 or PGF2 then the very limited
amounts of PGDH in these cells would allow
considerable quantities of these prostaglandins
to be produced. The lack of PGDH in these cells
suggests that very little or no PGE2 is
synthesised in choriocarcinoma cells.

Neutral steroids in urine were determined


quantitatively with gaschromatography on
capillary columns in a case of benign
hydatidiform mole associated with bilateral
theca-lutein cysts. A remarkable finding was the
very high levels of 17-hydroxypregnanolone
and pregnanetriol, which continued to rise until
the 15th day after molar evacuation.[Vanluchene
E et, al, 1977]

Answer: T, T, F, F, T

Rapid urine qualitative hCG assays may not be


reliable in the presence of markedly elevated
hCG levels found in molar pregnancy. [Davison
CM et al, 2004]
Urinary pregnanediol levels, on the other hand
are frequently decreased. [Clinical Laboratory
Medicine by Richard Ravel]
Decreased urinary pregnandeniol level but
increased in serum progesterone and estradiol17 beta suggest molar pregnancy

12

In Pursuit to Excel MCQ Exam for Professional III Examination (MCQ)


the nipples and mammary gland, as occurs
during nursing, leads to prolactin release. This
effect appears to be due to a spinal reflex arc
that causes release of prolactin-stimulating
hormones from the hypothalamus.

Prolactin
a) is secreted by the posterior pituitary
b) is necessary for mammary ductal
growth
c) level in plasma is unaffected by
smoking
d) is necessary for the establishment of
lactation
e) secretion is controlled by an inhibiting
factor

Estrogens provide a well-studied positive control


over prolactin synthesis and secretion. Increase
blood concentrations of estrogen during late
pregnancy appear responsible for the elevated
levels of prolactin that are necessary to prepare
the mammary gland for lactation at the end of
gestation.

Prolactin is a single-chain protein hormone


closely related to growth hormone secreted by
lactotrophs in the anterior pituitary. It is
synthesized as a prohormone.

Excessive
secretion
of
prolactin
hyperprolactinemia - is a relatively common
disorder in humans. This condition has
numerous causes, including prolactin-secreting
tumors and therapy with certain drugs.

Prolactin induces lobuloalveolar growth of the


mammary gland. Alveoli are the clusters of cells
in the mammary gland that actually secrete milk.
Prolactin stimulates lactogenesis or milk
production after giving birth. Prolactin, along
with cortisol and insulin, act together to
stimulate transcription of the genes that encode
milk proteins.

Common manifestations of hyperprolactinemia


in women include amenorrhea (lack of
menstrural cycles) and galactorrhea (excessive
or spontaneous secretion of milk). Men with
hyperprolactinemia
typically
show
hypogonadism, with decreased sex drive,
decreased sperm production and impotence.
Such men also often show breast enlargement
(gynecomastia), but very rarely produce milk.
http://www.vivo.colostate.edu/hbooks/pathphys/endo
crine/hypopit/prolactin.html

Dopamine serves as the major prolactininhibiting factor or brake on prolactin secretion.


It is secreted into portal blood by hypothalamic
neurons, binds to receptors on lactotrophs, and
inhibits both the synthesis and secretion of
prolactin. Agents and drugs that interfere with
dopamine secretion or receptor binding lead to
enhanced secretion of prolactin.

A minimum of 5 cigarettes significantly


decreases prolactin concentration in smokers. A
matched pairs comparison confirmed that
smoking reduces the level of prolactin [Gabriela
et al, 1995]
Answer: F, T, F, T, T

Prolactin secretion is positively regulated by


several hormones, including thyroid-releasing
hormone, gonadotropin-releasing hormone and
vasoactive intestinal polypeptide. Stimulation of
13

In Pursuit to Excel MCQ Exam for Professional III Examination (MCQ)


[Wikipedia] In pregnancy, liver produces more
transferrin.

With regards of the blood volume and its


composition during pregnancy

Serum Ferroxidase I and II are progressively


increased with serum Total Iron Binding
Capacity (TIBC) and unsaturated iron binding
capacity (UIBC) as pregnancy advances
[Agroyannis B et al]

a) the total RBC falls by about 20%


from the normal non-pregnant
volume
b) the packed cell volume falls
c) there is rise in the iron binding
capacity
d) the blood cholesterol rises
e) the protein bound iodine level falls

The total iron-binding capacity exceeded normal


starting in the sixth lunar month in the
nontreated group and in the seventh lunar month
in the treated group, and it decreased slightly in
the latter toward term and had returned to
normal in both groups by the fourth postpartum
week [George D. Malkasian]

Physiology of anemia in pregnancy


A disproportionate increase occurs in plasma
volume (25% to 50%) compared to red blood
cell mass (10% to 25%) during pregnancy, with
resultant hemodilution and reduction in
hematocrit of 3% to 5%. These changes begin at
approximately 6 weeks gestation and normalize
by 6 weeks postpartum. [The John Hopkins
Manual of Gynecology and Obstetrics 3rd Ed.]

Serum protein bound iodine levels increased


significantly with age in both sexes and in
pregnant
women,
while
it
decreased
significantly in alcoholics, cigarette smokers and
marijuana addicts [O M Ebuehi et al]
It is well known that with the effect of hormonal
changes during pregnancy, plasma lipid levels
increase. Expected elevations for triglyceride
and cholesterol levels during a normal
gestational period usually do not exceed 332
mg/dL
and
337
mg/dL,
respectively
(corresponding
95th
percentile
values).
However, elevations over the 95th percentile
values can be observed during pregnancy, and
patients with levels over these expected
adaptation levels can be divided into 2 groups:
(1)
supraphysiologic
hyperlipoproteinemia
during
pregnancy
and
(2)
extreme
hyperlipoproteinemia limited to gestational
period (triglyceride level >1000 mg/dL)
[Basaran A.]

Packed cell volume (PCV) is determined by


measuring the height of the red cell volume in a
micro-hematocrit capillary filled with whole
blood, after centrifugation. It is a directly
measured value, Meanwhile hematocrit (Hct) is
the corresponding calculated value (RBC X
MCV), PCV and Hct are interchangeable even
though PCV is slightly higher than the more
accurate Hct due to plasma trapping (between
the packed cells in a centrifuged capillary).
In present study hemoglobin percent and packed
cell volume was significantly decreased in 2nd
and 3rd trimester of pregnancy when compared
with the control group and same category of
women who were not supplemented with iron. It
is evident that the significantly low hemoglobin
percent and packed cell volume (PCV) in
pregnant women is due in part to dietary iron
deficiency. Therefore, iron therapy in pregnancy
is helpful to maintain the hemoglobin percent
and packed cell volume nearer to that of non
pregnant normal women. [Wahed F et al]

Answer: F, T, T, T, F

Total iron-binding capacity (TIBC) is a medical


laboratory test which measures the blood's
capacity to bind iron with transferrin.
14

In Pursuit to Excel MCQ Exam for Professional III Examination (MCQ)


become vigorous and the mesosalpinx contracts
to bring the tube in more contact with the ovary
while the fimbria contracts rhythmically to
sweep over the ovarian surface. As progesterone
level rises 4-6 days after ovulation, it inhibits
tubal motility. This may lead to relaxation of the
tubal musculature to allow passage of the ovum
into the uterus by the action of the tubal cilia.
The effects of estrogen and progesterone on
oviductal motility and morphology is mediated
through these steroids' receptors. The changes in
receptors levels are critical in determining the
functional state of the oviduct.[Diaa M. EIMowafi, MD;Zagagig University, Egypt]

Ovulation in human
a) is associated with surge of LH
b) is characteristically followed by the
development
of
secretory
endometrium
c) is associated with an increased in
motility of the Fallopian tube
d) is associated with a sustained fall in
the basal body temperature
e) followed by a rise in urinary
pregnanetriol

The midcycle LH surge is responsible for a


dramatic increase in local concentrations of
prostaglandins and proteolytic enzymes in the
follicular wall. These substances progressively
weaken the follicular wall and ultimately allow a
perforation to form. Ovulation most likely
represents a slow extrusion of the oocyte
through this opening in the follicle rather than a
rupture of the follicular structure [Novak
Gynecology 14 ed]

In women, ovulation causes an increase of onehalf to one degree Fahrenheit (one-quarter to


one-half degree Celsius) in basal body
temperature (BBT); monitoring of BBTs is one
way of estimating the day of ovulation. The
tendency of a woman to have lower
temperatures before ovulation, and higher
temperatures afterwards, is known as a biphasic
pattern. Charting of this pattern may be used as a
component of fertility awareness.[Wikipedia]
During the normal menstrual cycle the excretion
of pregnanetriol increased on the day that the
excretion of oestrone and oestradiol reached a
maximum during the follicular phase.
Pregnanediol excretion did not begin to increase
until 2 days later. The excretion of pregnanetriol
reached a peak and began to decrease before that
of pregnanediol. Thus the pattern of
pregnanetriol excretion was different from that
of the excretion of oestrogens or pregnanediol.
[K. Fotherby]
It is found that the cyclic change in urinary
pregnanetriol excretion is variable in extent but
there is a correlation between the urinary
excretion of oestrogens, pregnanediol and
pregnanetriol during the ovulatory cycle [Mary
TP & Ian FS]

Peristaltic movement of fallopian tube is


primarily regulated by three intrinsic systems:
the estrogen-progesterone hormonal milieu, the
adrenergic-nonadrenergic
system,
and
prostaglandins. Before ovulation, contractions
are gentle, with some individual variations in
rate and pattern. At ovulation, contractions

Answer: T, T, T, F, T

15

In Pursuit to Excel MCQ Exam for Professional III Examination (MCQ)


booking (1.55, 1.28 to 1.88), or maternal age
40 (1.96, 1.34 to 2.87, for multiparous women).
Individual studies show that risk is also
increased with an interval of 10 years or more
since a previous pregnancy, autoimmune
disease, renal disease, and chronic hypertension.
[Duckitt & Harrington]

Pre eclampsia is associated with


A) Diabetes Mellitus
B) Rhesus iso immunization
C) Urinary tract infection
D) Polyhydramnios
E) Twin pregnancy
Severe pre-eclampsia and eclampsia are
relatively rare but serious complications of
pregnancy, with around 5/1000 maternities in
the UK suffering severe pre-eclampsia and 5/10
000 maternities suffering eclampsia. In
eclampsia, the case fatality rate has been
reported as 1.8% and a further 35% of women
experience a major complication [RCOG
Guideline No. 10(A)]

The most significant risk factors for developing


pre-eclampsia are a history of pre-eclampsia and
the presence of Antiphospholipid antibodies.
[Duckitt & Harrington]
Pre-existing diabetes and a pre-pregnancy BMI
of 35 almost quadruple the risk; nulliparity, a
family history of pre-eclampsia, and twin
pregnancy almost triple the risk; and maternal
age 40, a booking BMI of 35, and a systolic
blood pressure 130 at booking double the risk.
[Duckitt & Harrington]

Who is at risk of getting pre-eclampsia?


1) First pregnancy
2) First pregnancy with a new partner
3) Aged 40 or over
4) Mother or sister had pre-eclampsia
during pregnancy
5) Pre-eclampsia in a previous pregnancy
6) Body mass index (BMI) of 35 or more
(you weigh 90 kg or more)
7) Multiple pregnancy
8) Medical problem such as high blood
pressure, kidney problems and/or
diabetes.
9) Pregnant from egg (oocyte) donation.

Pre-existing hypertension, renal disease, chronic


autoimmune disease, and 10 years between
pregnancies increase the risk but it is not clear
by how much. [Duckitt & Harrington]
These data demonstrates a significant positive
relation with maternal age, diabetes in
pregnancy, and fetal macrosomia with
polyhydramnios. Anemia during pregnancy,
cesarean delivery rate, and congenital anomalies
were significantly higher in the study
group.[Mathew Mariam et al]

Controlled cohort studies showed that the risk of


pre-eclampsia is increased in women with a
previous history of pre-eclampsia (relative risk
7.19, 95% confidence interval 5.85 to 8.83) and
in those with antiphospholipids antibodies (9.72,
4.34 to 21.75), pre-existing diabetes (3.56, 2.54
to 4.99), multiple (twin) pregnancy (2.93, 2.04
to 4.21), nulliparity (2.91, 1.28 to 6.61), family
history (2.90, 1.70 to 4.93), raised blood
pressure (diastolic 80 mm Hg) at booking
(1.38, 1.01 to 1.87), raised body mass index
before pregnancy (2.47, 1.66 to 3.67) or at

Answer: T, F, F, F, T

16

In Pursuit to Excel MCQ Exam for Professional III Examination (MCQ)


It is widely accepted that prostaglandins play a
central role in parturition through their actions
on uterine contractility and on cervical ripening.
In both human and nonhuman primates, the
major intrauterine sources of prostaglandins are
the fetal membranes and the deciduas [George J.
Haluska et al]. PGs have also been shown to
play an important role in up-regulation of the
fetal
hypothalamic-pituitary-adrenal
axis,
membrane rupture and the maintenance of
uterine and placental blood flow [McKeon &
Challis]

In normal labour
a) endogenous oxytocin is responsible
for the initiation of
uterine
contraction
b) there is progressive increase in
normal cortisol
c) the uterine contractions is increased
by
release
of
endogenous
prostaglandin
d) during the first stage of labour, the
maternal arterial pressure rises
during each uterine contraction

13, 14-dihydro-15-keto-prostaglandin F (PGFM)


levels increased a labor progressed, and reached
maximal levels before placental separation had
occurred. Peripheral plasma concentrations of
oxytocin did not change significantly at any
stage of labor or 2 hours post partum. These
results suggest that prostaglandins have a role in
the third stage of labor, and this finding may
have important clinical implications. [SM
Sellers et al]

They are four major hormonal systems involve


during labor which are oxytocin, endorphins,
epinephrine& Norepinephrine and prolactin.
Oxytocin causes the rhythmic uterine
contractions of labor, and levels peak at birth
through stimulation of stretch receptors in a
womans lower vagina as the baby descends.
The high levels continue after birth, culminating
with the birth of the placenta, and then gradually
subside. The baby also has been producing
oxytocin during labor, perhaps even initiating
labor; [Dr Sarah J Buckley MD]

With active labor there are clear identifiable


changes in arterial compliance and cardiac load
as reflected in rapid ejection times (RETs) and
pulse wave arrival times (PWATs), respectively.
These findings are absent in Prelabour. [Edward
H. Hon et al]

These results give further support to the


hypothesis that maternal stress leads to a
reduced concentration of prolactin and increased
concentration of cortisol whereas relief of pain
and maternal anxiety with meperidine lessens
both effects.[E. Onur et al]

Serial measurements of cardiac output and mean


arterial pressure were performed in 15 women
during the first stage of labour and at one and 24
hours after delivery.....Over the same period
basal
mean
arterial
pressure
also
increased......There were also further increases in
mean blood pressure during contractions.[S C
Robson et al]

Maternal plasma Cortisol levels in 62


primiparous women rose during labour and
remained high for 20 minutes after delivery.
Umbilical cord Cortisol levels were substantially
lower than maternal levels. The maternal
Cortisol level was slightly lower in those who
required oxytocin. This relationship between
maternal Cortisol levels and the use of oxytocin
was not affected by the use of epidural
analgesia. The mean maternal Cortisol level rose
to a lesser extent in the women who had epidural
analgesia than in those who did not.[Haddad &
Morris]

Answer: T, T, T, T, T

17

In Pursuit to Excel MCQ Exam for Professional III Examination (MCQ)


Regarding ectopic pregnancy
a) the common site is the cornu of uterus
b) anaemia is due to thrombocytopenia
c) hemoperitoneum results from tubal
rupture
d) ovary is another common site
e) shock may follow tubal rupture
Ectopic pregnancy refers to the implantation of a
fertilized egg in a location outside of the uterine
cavity, including the fallopian tubes, cervix,
ovary, cornual region of the uterus, and the
abdominal cavity. This abnormally implanted
gestation grows and draws its blood supply from
the site of abnormal implantation. As the
gestation enlarges, it creates the potential for
organ rupture because only the uterine cavity is
designed to expand and accommodate fetal
development. Ectopic pregnancy can lead to
massive hemorrhage, infertility, or death.
[Vicken P Sepilian] It is occur in 2% of all
pregnancy

Pictures from
http://emedicine.medscape.com/article/258768-overview

The classic clinical triad of ectopic pregnancy is


pain, amenorrhea, and vaginal bleeding (50%).
Patients may present with other symptoms
common to early pregnancy, including nausea,
breast fullness, fatigue, low abdominal pain,
heavy cramping, shoulder pain, and recent
dyspareunia.
38 year-old woman presented with gynaecologic
haemorrhage, pelvic pain and hypovolemic
shock. Without any ambiguity, the diagnosis
was directly made during contrast enhanced
Multidetector Computed Tomography (MDCT).
Massive hemoperitoneum with fresh blood clots
in the hypogastric area, active free peritoneal
extravasation of intravascular contrast material
and dramatic peripheral enhancement, - "ring of
fire" sign - of an adnexal cystic structure were
the key signs. [Coulier B et al]

Anything that impairs migration of embryo to


the andometrial cavity will predispose a woman
to ectopic pregnancy. this includes pelvic
inflammatory disease, history of prior ectopic
pregnancy, History of tubal surgery and
conception after tubal ligation, Use of fertility
drugs or assisted reproductive technology, Use
of an intrauterine device, Increasing age,
smoking, Salpingitis isthmica nodosum,
diethylstilbestrol (DES) exposure, a T-shaped
uterus, prior abdominal surgery, failure with
progestin-only contraception, and ruptured
appendix.

Finding of hemoperitoneum on ultrasound


examination may not be an absolute
contraindication to conservative management of
tubal ectopic pregnancy [Bignardi & Condous]
Rupture may be heralded by sudden, severe
pain, followed by syncope or by symptoms and
signs of hemorrhagic shock or peritonitis. Rapid
hemorrhage is more likely in ruptured cornual
pregnancies [The Merck Manuals]

Sites and frequencies of ectopic pregnancy. By


Donna M. Peretin, RN. (A) Ampullary, 80%;
(B) Isthmic, 12%; (C) Fimbrial, 5%; (D)
Cornual/Interstitial, 2%; (E) Abdominal, 1.4%;
(F) Ovarian, 0.2%; (G) Cervical, 0.2%.

Answer: F, F, T, F, T

18

In Pursuit to Excel MCQ Exam for Professional III Examination (MCQ)


or inadequacy of ovulation. Some studies say
that up to 50 percent of patients with
endometriosis have some degree of ovulatory
dysfunction and this should certainly be taken
into account in the treatment of patients with
endometriosis and infertility. [Endometriosis The '90s Outlook]

Endometriosis
a) commonly affects the ovaries
b) is usually not associated with
infertility
c) intestinal obstruction is a possible
complication if bowels are involved
d) often present with dysmenorrhea
e) is primarily an acute inflammatory
process

Usually, the pain associated with endometriosis


is right before or during the menstrual period in
the initial stages; however, as the disease
progresses, it may occur throughout the cycle.
The pain may be acute or chronic. In about half
of the patients with severe or extensive
endometriosis, the pain is chronic all through the
cycle which gets worse right before and during
menstruation, and during or shortly after
intercourse. [Endometriosis - The '90s Outlook]

Endometriosis is the presence of endometriallike tissue outside the uterine cavity, which
induces a chronic inflammatory reaction. It can
occur in various pelvic sites such as on the
ovaries, fallopian tubes, vagina, cervix, or
uterosacral ligaments or in the rectovaginal
septum. It can also occur in distant sites
including laparotomy scars, pleura, lung,
diaphragm, kidney, spleen, gallbladder, nasal
mucosa, spinal canal, stomach, and breast. [Ami
K Dav, MD]

Endometriosis may invade the rectovaginal


septum and the anterior rectal wall. It may also
involve the upper rectum and sigmoid colon,
infiltrating the muscularis. Cyclical rectal
bleeding (hematochezia) is pathognomonic of
endometriosis. However, transmural bowel
involvement by endometriosis remains a rarity.
The ileum, appendix, and cecum may also be
involved, leading to intestinal obstruction.
Cicatrization as a consequence of endometriosis
may lead to symptoms of obstruction even in
postmenopausal women. [Ami K Dav, MD]

Ovaries is the most common site for


implantation of the endometrial cell (up to 75%)
because 1) through the theory of retrograde
menstruation, ovaries are adjacent to the opening
of the tube in the pelvic area and that location
alone will make the ovaries more prone to be
contaminated with the regurgitated menstrual
flow.2) ovaries have the highest level of steroid
hormone compared to any other organ and hence
they represent an ideal environment for
implantation and growth of the endometrial
tissue.

Answer: T, F, T, T, F

Infertility in endometriosis are due to 1) increase


level of prostaglandin production which is
interfering with sperm-ovum interaction, embryo
growth, interfering with sperm motility, and
interfering with the function of some central
nervous system areas that are responsible for
control of reproduction and 2) The presence of
endometrial tissue in the pelvic and peritoneal
cavities will cause some degree of abnormality
19

In Pursuit to Excel MCQ Exam for Professional III Examination (MCQ)


alone have been described. The Cochrane
Database indicates that with the available
evidence,
antenatal
TRH
cannot
be
recommended for clinical practice at the present
time. Adverse maternal side-effects were
significant for women receiving antenatal TRH
and these included hypertensive episodes.

Question1: Which of the following statements


are true regarding the pharmacological induction
of fetal respiratory maturity?
a) Maternally administered thyroid releasing
hormone (TRH) alone may cause lung
maturation.
b) Antenatal TRH has no maternal side-effects.
c) Treatment with antenatal corticosteriods
between 28 and 32 weeks reduces the incidence
of fetal intraventricular haemorrhage.
d) The use of antenatal cortiosteriods between
28 and 32 weeks with pre-labour ruptured
membranes is associated with a significant
increase in neonatal infection.
e) Antenatal corticosteriods may cause longterm cognitive disabilities in infants treated
prenatally.

Q11: Beta sympathomimetic drugs:


a) Increase maternal stroke volume.
b) Decrease fetal heart rate.
c) Increase maternal bowel motility.
d) Decrease intracellular potassium in maternal
cells.
e) Decrease maternal urinary output.
Answer: T, F,F,F,T
Beta adrenergic agonists, currently the most
widely used tocolytic drugs in the UK, include
ritodrine, salbutamol and terbutaline. These
pharmacological agents act via Beta-1 and Beta2 adrenergic receptors. Beta-I receptor responses
include an increase in maternal heart rate and
stroke volume, a decrease in bowel motility and
an increase in metabolic lipolysis. The Beta-2
adrenergic response includes an increase in renal
renin production and a decrease in urinary
output.

a) False
b) False
c) True
d) False
e) False
The meta-analysis of 36 prospective studies
indicate that treatment between 28 and 32 weeks
with antenatal corticosteriods prior to the onset
of premature labour resulted in a substantial
reduction in the incidence of respiratory distress
syndrome. The reduction in respiratory
morbidity was associated with overall reductions
in the incidence of neonatal intraventricular
haemorrhage, necrotizing enterocolitis and early
neonatal death. There was no strong evidence of
any adverseeffects of corticosteroids in these
trials. Long-term follow-up of children in
several of these studies indicated no adverse
effect on growth, physical development or
cognitive skills.

Question, answer and discussion are taken from


Self-assessment questions: Premature labour, by
M.Kilby, Current Obstetrics & Gynaecology
(1996) volume 6 issue 3

Trials investigating the improved efficacy of


antenatal
TRH
given
with
antenatal
corticosteriods, as compared to corticosteriods
20

In Pursuit to Excel MCQ Exam for Professional III Examination (MCQ)


significantly
reduces
intraventricular
haemorrhage.
d) Antibiotic administration significantly
reduces maternal infective morbidity in
prelabour ruptured membranes.
e) Intrapartum antibiotic therapy significantly
reduces fetal infective morbidity in prelabour
ruptured membranes.

Question 2: The following are contraindications


to tocolytic inhibition of preterm labour:
a) Abruptio-placenta.
b) Chorioamnionitis.
c) Pre-labour ruptured membranes.
d) Severe pregnancy induced hypertension.
e) Mild intrauterine growth restriction.

a) True
b) False
c) False
d) True
e) True

a) True
b) True
c) False
d) True
e) False

Analgesia and anaesthesia of choice for preterm


labour and delivery are not well established.
Epidural anaesthesia may well have benefits
especially in the management of the first and
second stage of a preterm vaginal breech
delivery. However, such a choice of analgesia is
contraindicated in chorioamnionitis. Although
both an elective episiotomy and low forceps
delivery have been historically discussed as
useful in reducing the length of second stage and
protecting the premature fetus at delivery, there
are no data to support this as a potential benefit
to the fetus.

Antepartum haemorrhage of whatever cause is a


contraindication to the suppression of preterm
labour as is intrauterine infection and severe
pregnancy induced hypertension which may be
life threatening to the mother. In prelabour
rupture of membranes, although there is no
absolute contraindication to tocolysis as long as
there
is
no
overt
evidence
of
chorioamnionitis,there is little evidence to
suggest that tocolysis significantly improves
perinatal mortality. In selected cases of mild
intrauterine growth restriction in which the fetus
is not compromised, it may be beneficial to
suppress
labour
to
allow
antenatal
corticosteriods to be administered.

Administration of antibiotics prophylactically to


women with prelabour ruptured membranes
preterm delays delivery and reduces both
maternal and neonatal infection. No effect has
yet been shown demonstrating an improvement
in perinatal mortality. In particular, in women
known to be carrying Group B Streptococci,
intrapartum antibiotics should be adopted as
standard care.

However, any evidence of fetal compromise


should lead to prompt delivery of the fetus.

Question3: During a preterm delivery:


a) Epidural anaesthesia is contraindicated if
chorioamnionitis is present.
b) Elective episiotomy is advised for all vaginal
deliveries.
c) The elective use of low outlet forceps delivery

Question, answer and discussion are taken from


Self-assessment questions: Premature labour, by
M.Kilby, Current Obstetrics & Gynaecology
(1996) volume 6 issue 3

21

In Pursuit to Excel MCQ Exam for Professional III Examination (MCQ)


There is no evidence that corticosteriod
administration suppresses preterm labour and
there is anecdotal evidence indicating that intraamniotic injection of betamethasone may
actually be used to induce labour.

Q4: The following drugs significantly reduce


preterm labour when administered:
a) Ritodrine.
b) Indomethacin.
c) Nifedipine.
d) Pethidine.
e) Dexamethasone.

Q5: The following are useful predictors of


preterm labour:
a) A previous history of preterm labour.
b) Maternal plasma oestradiol concentration 14
days prior to onset of labour.
c) Fetal fibronectin.
d) Multiple pregnancy.
e) Uterine anomaly.

ANSWER 4
a) True
b) False
c) False
d) False
e) False

Answer: T, F, T, T, T
Although beta-sympatho-mimetics themselves
may not have beneficial effects for the fetus they
are effective at postponing delivery, especially
for short intervals of up to 24 h. This suggests
that they may have a place if, for instance,
corticosteriods could be administered to promote
pulmonary maturity. However, injudicious use
of corticosteriods and beta-sympathomimetics
have been reported to precipitate acute
pulmonary oedema.
The only statistically
significant differences in prospective trials
comparing the use of calcium antagonists with
beta-sympatho-mimetics have indicated that
there were fewer neonates of less than 2.5 kg
and there were more admissions to neonatal'
intensive care after the treatment with calcium
antagonists. At the present time, only a small
number of prospective studies are reported.
Meta-analysis from the Cochrane Database does
not support the use of calcium antagonists or
magnesium sulphate in preference to betasympatho-mimetics in the suppression of
preterm labour.

The background rate of preterm labour in the


UK is approximately 7%. If a patient has had a
previous preterm labour this rises to between 14
and 18%. A uterine anomaly is proven in
between 5 and 16% of all preterm deliveries.
Multiple pregnancy is associated with preterm
delivery and this is paticularly true with
monochorionic placentation. The majority of
biochemical prediction indices, including serum
oestradiol concentrations, have a poor predictive
index at detecting preterm delivery. Fetal
fibronectin, a component of the extra cellular
matrix in the cervix, has been shown both in
cross sectional studies, and more recently in
longitudinal studies, to have a positive predictive
value of preterm delivery in high risk patients of
46%.
Question, answer and discussion are taken from
Self-assessment questions: Premature labour, by
M.Kilby, Current Obstetrics & Gynaecology
(1996) volume 6 issue 3

Pethidine may suppress both fetal and maternal


respiration and has no proven benefit in the
management of preterm labour.

22

In Pursuit to Excel MCQ Exam for Professional III Examination (MCQ)


c) True
d) True
e) False

Q6: Regarding the lecithin-sphingomyelin ratio:


a) This test is carried out on amniotic fluid.
b) Sphingomyelin is a general membrane lipid.
c) The ratio for normal pregnancies is greater
than 0.5 at 20 weeks.
d) If the ratio is less than 2 there is less than a
10% chance of the baby developing respiratory
distress syndrome.
e) Lecithin is produced by type II pneumocytes.

Cervical incompetence is a dysfunction in the


integrity ofthe internal cervical os. It is
characterised by painless cervical dilatation in
the mid second trimester. With regard to
aetiology, several factors are important. In-utero
exposure of Diethylstilboestrol carries a 45%
risk of pregnancy loss due to cervical
incompetence. Acquired factors include cervical
trauma. This includes over zealous mechanical
dilatation of the cervix prior to diagnostic
curettage. Loop excision of the transformation
zone is rarely associated with cervical
incompetence. With contemporary cervical
cerclage, the majority of patients have their
operation during pregnancy. The classical
Shirodkar's suture and McDonald's suture have
approximately the same success rate in
prevention of premature labour. Ritodrine alone
will not stop the contractions due to cervical
incompetence and there is little prospective
evidence to confirm that ritodrine therapy has a
role to play perioperatively in cervical cerclage.

Answer: T, T, F, F, T
The L/S ratio was introduced by Gluck in 1971.
Amniocentesis allows the collection of fluid and
the results are expressed as the ratio of lecithin
(phosphatylcholine) fraction enriched by cold
acetone precipitation. The sphingomyelin is used
as a control because amniotic fluid volume
changes during gestation cannot be accurately
measured clinically. In normal pregnancies the
L/S ratio is less than 0.5 at 20 weeks. The value
of 2.0 indicates a low risk of respiratory distress
syndrome at any point in gestation. The L/S ratio
is not reliable if amniotic fluid is heavily
contaminated with blood or meconium. Lecithin
and other phosphatyl-lipids are produced by the
type II pneumocytes within the lungs.

Question, answer and discussion are taken from


Self-assessment questions: Premature labour, by
M.Kilby, Current Obstetrics & Gynaecology
(1996) volume 6 issue 3

Q7: With cervical incompetence:


a) In-utero exposure to Diethylstilboestrol (DES)
is a causative factor.
b) A history of loop diathermy excision of the
transformation zone of the cervix for treatment
of CIN is a common association.
c) Shirodkar and McDonald cerclage techniques
carry the same success rates.
d) When the amniotic membranes are ruptured
then cervical cerclage is contraindicated.
e) This may be treated by oral Ritodrine.
ANSWER 7
a) True
b) False
23

In Pursuit to Excel MCQ Exam for Professional III Examination (MCQ)


QUESTION 8: In preterm labour complicated
by a low-grade infection:
a) Bacterial phospholipases cause a direct
release of arachidonic acid from the amnion and
decidual cells.
b) Bacterial lipopolysaccharides stimulate
increased production of interleukin-I (1L-I) from
amnion and decidual cells.
c) IL-1 stimulates the decidual production of IL8
d) Corticotrophin releasing hormone (CRH)
does not stimulate prostaglandin production
from decidual cells.
e) Annexin production by the trophoblast and
decidua decrease with the onset of labour.

syncytio-trophoblast of patients with preterm


labour.

ANSWER: T, T, T, F, T

Experimental animal models have suggested that


Relaxin has an inhibitory effect on myometrial
activity. This is separate from that induced by
progesterone. The inhibitory effect of Relaxin is
more rapid in onset than that of progesterone.
Although spontaneous myometrial contractility
is suppressed, sensitivity to oxytocin is
maintained and its primary effects are one of
frequency modulation of contractions. Relaxin
also elevates uterine cyclic AMP production in
common with other uterine relaxants. It is
unknown whether Relaxin has its effect directly
on the myometrium, but experimental evidence
indicates that cultured myometriai cells, when
stimulated with Relaxin, produce prostacyclin.

Q9: Relaxin:
a) Has a stimulatory effect on the myometrium.
b) Has an inhibitory effect that is more rapid
than progesterone.
c) Affects frequency modulation of uterine
contractions.
d) Elevates uterine cyclic AMP concentrations.
e) Stimulates prostocyclin production by the
myometrium.
Answer: F, T, T, T, T

Approximately 30% of all preterm labours are


associated with some mild chorioamnionitis.
Bacterial phospholipases directly release
arachidonic acid which are precusors of many
eieosanoids from decidual cells. Also
lipopolysaccharides stimulate decidual cells to
increase cytokines such as IL-I and tumour
necrosis factor from amnion and decidual cells.
The cytokines are elevated in the amniotic fluid
from patients with preterm labour and infection
and may secondarily release other cytokines
such as IL-8 which are chemotaxtic peptides for
neutrophils and T-cells. Cortieotrophin releasing
hormone is produced by the syncytio-trophoblast
and stimulates prostaglandin production from
cultured decidual cells. The prostaglandins
produced may have a direct effect upon the
myometrium which is adjacent to the chorion,
amnion and decidual cells.The annexins are
substrates for tyrosine kinases and influence
phospholipase A2 production and, therefore,
may increase myometrial intracellular free
calcium concentration. It is believed that they
bind to phospholipids preventing phospholipase
A2 from gaining access to its substrate. Both
annexin protein and mRNA are decreased in the

Question, answer and discussion are taken from


Self-assessment questions: Premature labour, by
M.Kilby, Current Obstetrics & Gynaecology
(1996) volume 6 issue 3

24

In Pursuit to Excel MCQ Exam for Professional III Examination (MCQ)


Q5: Concerning placenta praevia, which of the
following statements is/are true:
a) The incidence of placenta praevia is 1 in
5000.
b) Placenta praevia is more common in
Caucasian women.
c) Owing to the position of the placenta in
placenta praevia, prolapsed cord is much less
likely.
d) Previous lower segment Caesarean section is
a risk factor for placenta praevia.
e) There is no association between the incidence
of placenta praevia and maternal age.

Q7: which of the following statements


concerning placental abruption is/are true:
a) In western society placental abruption is
commonly associated with poor nutrition.
b) Placental abruption can occur in association
with external cephalic version.
c) Placental abruption is more common with
multiple pregnancy.
d) Placental abruption is more common with a
history of previous abruption.
e) Postpartum haemorrhage is a common
complication of placental abruption, which often
requires hysterectomy to control the bleeding.

a) False. The true incidence is 0.3-1.0%.


b) False. Placenta praevia is more common in
black women.
c) False. Prolapsed cord is up to three times
more likely with placenta praevia.
d) True.
e) False. More common in older women and
multiparous women.

a) False. Poor nutrition has previously been


reported as being associated with placental
abruption, but within the western world this is
now a rare finding.
b) True. External cephalic version is associated
with placental abruption, especially if the
manoeuvre is carried out under general
anaesthetic. Presumably, if the manoeuvre is
carried out under general anaesthetic, the
operator exerts more 'energy' into the procedure
as she/he does not have the benefit of
appreciating the maternal perception of
discomfort.
c) True.
d) True.
e) False. Postpartum haemorrhage is a common
sequel of placental abruption, but only rarely is
hysterectomy required for its control. Senior
obstetric input is mandatory in the management
of postpartum haemorrhage under these
circumstances, as it is of the utmost importance
that a hysterectomy is performed neither too
early nor too late.

Question, answer and discussion are taken from


Self-assessment questions: The Placenta, by
S.Smith, Current Obstetrics & Gynaecology
(1998) volume 8 issue 1

25

In Pursuit to Excel MCQ Exam for Professional III Examination (MCQ)


Question, answer and discussion are taken from
Self-assessment questions: Recurrent Abortion
by H.J.A. Carp, Current Obstetrics &
Gynaecology (1999) volume 9 issue 1

Question 4: Anti phospholipid syndrome:


a) The mechanism of action of antiphospholipid
antibodies is unknown.
b) In the presence of antiphospholipid
antibodies, pregnancy losses occur with equal
frequency in all three trimesters.
c) The treatment of choice is with steroids and
aspirin.
d) Anticardiolipin antibody is a very specific
marker of pregnancy loss.
Answer: T, F, F, F, T
Various pathways have been suggested to
explain the action of antiphospholipid
antibodies: coagulation in small vessels leading
to the placenta, vasoconstriction owing to
inhibition of prostacycline, and disruption of the
phospholipid intracellular bridges between
elements of trophoblast.
In the presence of antiphospholipid antibodies,
approximately 60% of pregnancy losses will be
in the first trimester, but there is a heavy
preponderance (40%) of second- and thirdtrimester losses. Various treatment regimens
have been used. At present heparin (or lowmolecular-weight heparin) and low-dose aspirin
seems to be the most effective regimen. Steroids
are no longer widely used owing to the possible
side effects. However, the issue is far from
settled.
Anticardiolipin antibody is a very non-specific
marker of pregnancy loss. It can be raised after
viral infections etc. It is only of real significance
if a high level is present, or if thromboses and/or
other autoantibodies are present.
Treatment of thrombophilias, has mainly been
found to be effective in cases of late pregnancy
loss rather in first trimester miscarriage.

26

In Pursuit to Excel MCQ Exam for Professional III Examination (MCQ)


Question 2: Concerning the epidemiology of
preeclampsia:
a) The incidence of preeclampsia in primigravid
mothers in the UK is 1%.
b) Preeclampsia is more common in the black
population.
c) The incidence of preeclampsia is higher in
triploid pregnancy.
d) The incidence of preeclampsia is higher in
lower socio-economic groups.
e) The risk of developing preeclampsia is
decreased in a pregnancy complicated by
placenta praevia.

Question 1: Regarding the diagnosis and


classification of hypertension in pregnancy:
a) The phase 5 Korotkoff sound is more reliably
detected in pregnancy than the phase 4 sound.
b) Significant proteinuria is denoted by the
presence of 500 mg of protein in a 24 h
collection.
c) The combination of hypertension and
proteinuria always signifies the presence of
preeclampsia.
d) The presence of oedema is a useful diagnostic
sign.
e) Preeclampsia never presents before 20 weeks
gestation.

Answer: F, T, T, T, T
The incidence of pre-eclampsia in privigravid
women in the UK is between 7 and 10%. Some
ethnic groups appear to be at increased risk and
in particular the black population is said to have
an increased risk. It is, however, difficult to
separate this risk from other predisposing factors
such as parity, obesity and an inherited tendency
to essential hypertension. There is a welldocumented
relationship
between
hyperplacentosis and the development of preeclampsia; thus, multiple, molar and triploid
pregnancy are all associated with an increased
incidence of pre-eclampsia. Placenta praevia is
traditionally said to confer a decreased risk of
developing pre-eclampsia and certainly the
incidence of one is lower in the presence of the
other. However, the protective effect of placenta
praevia may merely reflect the increased
incidence of earlier delivery associated with this
condition. A higher incidence of pre-eclampsia
has been reported in lower socio-economic
groups. This difference most likely reflects
differences in age, parity, levels of antenatal
care and smoking.

Answer: T, F, F, F, F
Pre-eclampsia is generally defined as
hypertension of at least 140/90 mmHg measured
on at least two separate occasions and arising de
novo after the 20 weeks gestation, in the
presence of 300 mg of protein in a 24 h
collection of urine.While the rapid appearance of
oedema (particularly when it involves the face)
may herald the advent of pre-eclampsia, this
sign is not part of the diagnostic pattern and,
indeed, is present in two-thirds of normal
pregnant women. The combination of
hypertension and proteinuria does not always
signify preeclampsia. Renal disease often
presents with proteinuria and may or may not be
accompanied by hypertension. This is the more
likely diagnosis if the presentation is before 20
weeks gestation. However, a hydatidiform mole
can rarely present in the first trimester with
preeclampsia. The measurement of blood
pressure in pregnancy has long been the subject
of much controversy. The evidence at present
suggests that Korotkoff phase V sound is the
most reproducible endpoint in pregnancy.

Question, answer and discussion are taken from


Self-assessment questions: The pathogenesis of preeclampsia, by L.C. Kenny, Current Obstetrics &
Gynaecology (1999) volume 9 issue 4

27

In Pursuit to Excel MCQ Exam for Professional III Examination (MCQ)


d) Decreased maternal levels of vitamin E.
e) An increase in the maternal ratio of
prostacyclin to lipid peroxides.

Question 3: Pre-eclampsia is associated with:


a) An increase in the maternal platelet count.
b) An increase in the maternal mean platelet
volume.
c) A decrease in maternal thromboxane
production.
d) An increase in circulating maternal levels of
bthromboglobulin.
e) An increase in platelet adhesion.

Answer: F, F, T, T, F
There are many emerging similarities between
the condition of atherosclerosis and preeclampsia. The two conditions share a similar
lipid profile; low maternal maternal serum
concentrations of HDL cholesterol, raised
concentrations of serum triglycerides, and
increased formation of small, dense LDL
particles. Furthermore, many of the risk factors
for the two disorders are similar; obesity, black
race, lipid abnormalities, insulin resistance and
raised
homocysteine
concentrations
all
predispose to atherosclerosis and pre-eclampsia.
These similarities and the generally accepted
role of oxidative stress in atherosclerosis,
support the emerging concept that reduced
placental perfusion interacts with maternal
factors to generate oxidative stress in preeclampsia. The most important lipid-soluble
anti-oxidant in human plasma is vitamin E. In
normal pregnancy plasma levels of prostacyclin
and vitamin E increase, whereas thromboxane
levels are decreased and serum levels of lipid
peroxide remain relatively constant. In preeclampsia, there is an imbalance in the
thromboxane to prostacyclin ratio with elevated
maternal levels of lipid peroxides and decreased
levels of vitamin E.

Answer: F, T, T, T, T
Longitudinal analysis of the peripheral platelet
count in pregnancy has revealed that preeclampsia is associated with thrombocytopaenia.
As a result the mean platelet volume has been
reported to increase in preeclampsia. The
population of larger platelets in this condition
may be explained by increased consumption,
leading to an increase in the proportion of
immature, larger platelets in the peripheral
circulation. There is substantial evidence of
platelet activation in pre-eclampsia. Circulating
levels of factors stored within platelets reflect
platelet activation Several studies have reported
increased levels of the platelet granule protein bthromboglobulin in women with pre-eclampsia
as compared with normal pregnant controls and
this elevation precedes the development of
clinical signs by at least 4 weeks. Thromboxane
production, as measured by urinary metabolites,
is increased in women with pre-eclampsia. This
increase in thromboxane production leads to an
increase in platelet adhesion and aggregation
.
Question 4: Pre-eclampsia is associated with:
a) Increased maternal serum concentrations of
highdensity
lipoprotein cholesterol.
b) Decreased maternal concentrations of serum
triglycerides.
c) Increased maternal serum homocysteine
concentrations.

Question, answer and discussion are taken from


Self-assessment questions: The pathogenesis of
pre-eclampsia, by L.C. Kenny, Current
Obstetrics & Gynaecology (1999) volume 9
issue 4

28

In Pursuit to Excel MCQ Exam for Professional III Examination (MCQ)


Question 8: Liver function:
a) Normal pregnancy is associated with a
decrease in maternal alkaline phosphatase.
b) During normal pregnancy maternal levels of
alanine transaminase (ALT) and aspartate
transaminase (AST) fall.
c) The principal pathological finding in the liver
in pre-eclampsia comprises of periportal fibrin
deposition, haemorrhage and hepatocellular
necrosis.
d) An increase in maternal levels of
transaminases
in
pre-eclampsia
reflects
hepatocellular damage.
e) Lactate dehydrogenase levels are a reliable
marker of haemolysis.

Question 5: Maternal cardiovascular changes


associated with preeclampsia include:
a) An increase in maternal plasma volume.
b) A decrease in the maternal haematocrit.
c) A rise in the maternal plasma oncotic
pressure.
d) An exaggerated maternal arterial response to
angiotensin II.
e) An exaggerated maternal arterial response to
bradykinin.
Answer: F, F, F, T, F
In normal pregnancy, plasma volume increases
by about 40% and leads to a fall in the
haematocrit. The traditional and most widely
accepted model of the haemodynamic
characteristics of pre-eclampsia is one of a
relatively
reduced
plasma
volume,
vasoconstriction and resulting hypoperfusion of
organs such as the placenta and kidneys. Thus,
the haematocrit in pre-eclampsia is typically
increased. The reduced plasma volume
associated with pre-eclampsia reflects the shift
of fluid into the extravascular space across leaky
capillary membranes. Increasing proteinuria in
pre-eclampsia
leads
to
marked
hypoalbuminaemia and a fall in plasma oncotic
pressure. In normal pregnancy, a relative arterial
refractoriness to angiotensin II develops. This
can be detected as early as 8 weeks and is
maximal at term. In women who develop preeclampsia, the pressor response to angiotensin II
remains relatively greater than in normal
women. Ex vivo studies have demonstrated that
endothelium- dependent relaxation to numerous
substances, including bradykinin, is impaired in
arteries isolated from women with preeclampsia. The combination of these two
findings may contribute to the relative
vasoconstriction associated with pre-eclampsia.

Answer: F, T, T, T, T
The relative haemodilution of normal pregnancy
leads to an approximate 20% reduction in the
level of circulating liver enzymes in pregnancy,
with the exception of alkaline phosphatase,
which normally increases with increasing
gestation. Abnormal liver function in preeclampsia reflects liver dysfunction occurring as
a result of vasoconstriction of the hepatic bed.
Elevated levels of transaminases reflect
heptocellular necrosis. Haemolysis may increase
asparate
transaminase,
but
will
disproportionately
increase
lactate
dehydrogenase levels with serial measurements
providing a reliable indicator of the degree of
haemolysis present. The histopathology of the
liver in pre-eclampsia comprises peri-portal
fibrin deposition, haemorrhage and heptocellular
necrosis. It is thought that segmental hepatic
vasospasm leads to localized coagulopathy
allowing fibrin deposition while endothelial and
liver-cell necrosis produce haemorrhage.
Question, answer and discussion are taken from
Self-assessment questions: The pathogenesis of preeclampsia, by L.C. Kenny, Current Obstetrics &
Gynaecology (1999) volume 9 issue 4

29

In Pursuit to Excel MCQ Exam for Professional III Examination (MCQ)


levels of fibrinopeptides in women with preeclampsia as compared with normal pregnant
women. Anti-thrombin III is produced by the
liver and is an important inhibitor of
coagulation. Levels are unchanged in normal
pregnancy but are decreased in the majority of
women with pre-eclampsia. The decline in antithrombin III activity is thought to result from
increased consumption and has been reported to
precede the development of clinical signs by as
much as 13 weeks. Protein C is a potent
inhibitor of activated factor V and VIII, and is
an activator of fibrinolysis. Levels are
substantially
reduced
in
pre-eclampsia.
Activated protein C resistance resulting from a
mutation in coagulation factor V has now
emerged as a leading cause of thrombosis in
pregnancy and a recent report links the HELLP
syndrome with factor V mutation. Protein S
serves as a cofactor for activated protein C.
Levels of protein S in pregnancy may decrease
to levels similar to those with congenital protein
S deficiency. Furthermore, levels of protein S
decrease further in women with pre-eclampsia,
as compared with normal pregnant controls.

Question 7: Changes in the coagulation system


in pre-eclampsia include:
a) An increase in maternal levels of antithrombin
III.
b) An increase in maternal levels of factor VIII
related antigen.
c) A decrease in maternal levels of
fibrinopeptide A.
d) A decrease in maternal levels of protein C.
e) An increase in maternal levels of protein S.
Answer: F, T, T, T, F
In normal pregnancy, the overall state of the
coagulation system is one of activation.
Evidence for this comes from studies which
report increased concentrations of clotting
factors and raised levels of highmolecularweight fibrinogen complexes. Sensitive studies
of coagulation factors provide evidence that
pre-eclampsia
accentuates
a
state
of
hypercoagulability already created in normal
pregnancy. During normal pregnancy, the levels
of factor VIII coagulation activity and factorVIII-related antigen show a proportional rise
and, thus, their ratio remains constant. In preeclampsia, there is an early rise in the factorVIII-related antigen:coagulation activity ratio
which correlates with the severity of the disease.
This increased ratio is a result of increased
levels of factor- VIII-related antigen. This
substance is synthesized by endothelial cells and
megakaryocytes and is released by aggregating
platelets. It, therefore, seems probable that the
increase in circulating levels is a result of
endothelial damage and platelet aggregation.
The action of thrombin on fibrinogen is a crucial
step in the coagulation cascade. Thrombin
cleaves two pairs of peptides, fibrinopeptides A
and B from fibrinogen to produce fibrin.
Fibrinopeptide concentrations are considered to
be the best markers of accelerated thrombosis.
The majority of studies have reported increased

Question, answer and discussion are taken from


Self-assessment questions: The pathogenesis of
pre-eclampsia, by L.C. Kenny, Current
Obstetrics & Gynaecology (1999) volume 9
issue 4

30

In Pursuit to Excel MCQ Exam for Professional III Examination (MCQ)


Question 10: Eclampsia:
a) Is an absolute indication for delivery.
b) Complicates 15 in 10 000 maternities in the
UK.
c) Can occur in the absence of significant
hypertension.
d) Leads to death most commonly by cerebral
haemorrhage.
e) Can result in cortical blindness.

Question 6: Concerning renal function in preeclampsia:


a) The proteinuria of pre-eclampsia results
mainly from a loss of intermediate weight
proteins.
b) The rate of increase in the amount of
proteinuria is both a sensitive and specific
predictor of maternal outcome.
c) Glomerular filtration deteriorates first and is
followed by a deterioration in tubular function.
d) Hyperuricaemia generally develops before
proteinuria.
e) Glomerular endotheliosis is present in all
cases of established pre-eclampsia.

Answer: T, F, T, T, T
The principal and clinically most disturbing
cerebral sequelae of pre-eclampsia are eclamptic
convulsions. Eclampsia is defined as the
occurrence of one or more convulsions, not
attributable to other cerebral conditions in a
patient with pre-eclampsia. It should be noted,
however, that eclampsia can rarely present in the
absence of significant hypertension. The
reported incidence varies geographically; figures
from the UK suggest the incidence to be 4.9/10
000 maternities. Eclampsia is thought to result
from
focal
cerebral
vasospasm
and
hypoperfusion leading to abnormal electrical
activity and seizures. The lack of any
neurological deficit following uncomplicated
eclampsia supports the concept that vasospasm
precipitates the convulsions. This is in contrast
to convulsions caused by thrombosis and
haemorrhage, as is often the case in postpartum
eclampsia, when significant cortical damage
may occur, including the development of
cortical blindness. The commonest cause of
death in women dying with eclampsia is cerebral
haemorrhage.

Answer: T, F, F, T, F
Renal function deteriorates in pre-eclampsia in
two stages. The first stage involves impairment
of tubular function and this is reflected by a
reduction in uricacid clearance and the
development
of
hyperuricaemia.
Later,
glomerular filtration becomes impaired and
proteinuria of intermediate selectivity develops.
An increasing plasma urate is thus an early sign
in the evolution of pre-eclampsia and proteinuria
is conventionally recognized as a late sign. The
presence of proteinuria heralds a poorer
prognosis for the fetus, but the actual amount of
proteinuria and the rate of increase have been
found to be poor predictors of maternal or
perinatal outcome. Renal biopsy in established
cases of pre-eclampsia demonstrates a
characteristic
non-inflammatory
lesion
commonly
referred
to
as
glomerular
endotheliosis. This primarily involves a
combination of swelling of the glomerular
endothelial cells and sub-endothelial fibrinoid
deposits, which encroach on and occlude the
capillary lumen. This lesion is characteristic, but
not pathognomonic of pre-eclampsia, as normal
renal pathology has been described in some
cases.

Question, answer and discussion are taken from


Self-assessment questions: The pathogenesis of preeclampsia, by L.C. Kenny, Current Obstetrics &
Gynaecology (1999) volume 9 issue 4

31

In Pursuit to Excel MCQ Exam for Professional III Examination (MCQ)


intrauterine growth retardation, oliguria, renal
failure and neonatal anuria in late pregnancy,
and are thus contraindicated in pregnancy.

Question 9: Regarding drug treatment in preeclampsia:


a) Angiotensin-converting enzyme inhibitors are
not associated with serious side effects in the
antenatal period.
b) Peak plasma levels of methyldopa are reached
24 h after an oral dose.
c) Labetolol is a combined a- and badrenoceptor blocker.
d) Nifedipine can be safely used concomitantly
with magnesium sulphate.
e) Hydrallazine has no effect on uteroplacental
blood flow.

Question, answer and discussion are taken from


Self-assessment questions: The pathogenesis of preeclampsia, by L.C. Kenny, Current Obstetrics &
Gynaecology (1999) volume 9 issue 4

Answer: F, F, T, F, F
The hypertension of pre-eclampsia is caused by
increased peripheral resistance and drug
treatment is directed towards relieving this
without
compromising
cardiac
output.
Methyldopa has been extensively studied in
pregnancy and is the agent of choice for chronic
blood-pressure control. It reduces systemic
vascular resistance without significantly altering
heart rate or cardiac output. Peak plasma
concentrations are reached after 2 h, although
the maximum fall in arterial pressure occurs 48
h after an oral dose. Labetolol is a combined aand b-adrenoreceptor blocker. The a1 blockade
induces vasodilatation with little change in the
cardiac output. Nifedipine can be used for acute
or chronic treatment and appears to be safe in
pregnancy. It lowers systemic vascular
resistance and improves cardiac output without
adversely comprising uteroplacental blood flow.
It is, however, best avoided in combination with
magnesium sulphate as profound hypotension
may result. Hydrallazine is used for the acute
management of hypertensive crises. It too lowers
systemic vascular resistance, but it has a variable
effect on uteroplacental blood flow and can
occasionally lead to fetal distress. Angiotensinconverting enzyme inhibitors are associated with
32

Das könnte Ihnen auch gefallen